MSK/Rheumatology Practice Q's (Exam 2)

Pataasin ang iyong marka sa homework at exams ngayon gamit ang Quizwiz!

The _____________ are the parts of the body most commonly affected by polymyositis.

shoulders and hips

Patients with ankylosing spondylitis show ______________ on radiography.

vertebral fusion (bamboo spine)

Parotid gland enlargement, keratoconjunctivitis sicca, and xerostomia are hallmarks of Sjogren's disease. What does Sjogren's disease put patients at risk for? a. multiple myeloma b. lymphoma c. leukemia d. bone cysts

b. lymphoma

Spondylolysis is a defect or injury in the pars interarticularis of the vertebral arch, most often at the ________________ vertebrae.

4th or 5th lumbar

Which of the following options are labs that should be done every 3 months for MTX monitoring? a. CBC b. LFT's c. CXR d. kidney ultrasound

A & B should be done every 3 months CXR should be done at baseline and if possible pulmonary fibrosis. Kidney ultrasound not indicated

Which of the following drugs for an acute gout attack are safe in pregnancy? (multiple answers may apply) a. glucocorticoids b. NSAIDs c. colchicine

A and B apply (glucocorticoids and NSAIDs) Colchicine is NOT safe in pregnancy

Which gastrointestinal pathogen is classically associated with reactive arthritis? Campylobacter jejuni Chlamydia trachomatis Staphylococcus aureus Streptococcus pneumoniae

Campylobacter jejuni Reactive Arthritis Patient with a history of recent GI or Chlamydia infection Complaining of acute asymmetric arthritis PE will show conjunctivitis, arthritis, urethritis Labs will show HLA-B27 Treatment is NSAIDs Comments: Can't see (conjunctivitis/uveitis), can't pee (urethritis), can't climb a tree (arthritis)

A patient with a + Hoffman sign, + Lhermitte sign, and a + Rhomberg test may be diagnosed with ___________ ____________.

Cervical myelopathy Compression of the cervical spinal cord Sxs: stiffness, diffuse, paresthesia, radiculopathy Increased reflexia, wide based gait, grip strength weakness Tx: Surgical decompression +/- instrumentation Common in elderly, asian descent

"Popcorn" appearance on Xray would point you towards which malignant bone tumor?

Chondrosarcoma -Assoc w/ osteochondromas -Local swelling and pain +/- patho fracture Bone biopsy required

Metastasis to bone typically spreads via (lymphatic/hematogenous) _____________ spread or local invasion.

Hematogenous spread Breast/lung -> t-spine d/t venous drainage through azygous vein -> plexus of Batson in t-spine Prostate: L/S spine & pelvis -> pelvic plexus in lumbar region

A (T1/T2) _____ weighted MRI shows the best anatomy and bone marrow.

T1-weighted Black (low intensity) -Fluid (urine/CSF) Gray (intermittent intensity) -Muscle -Gray matter White (high intensity) -Fat -White matter

Which of the following special populations in leflunomide contraindicated in? A. Pregnancy B. Breastfeeding C. Renally impaired D. Hepatically impaired E. All of the above

E. All of the above Avoid pregnancy until undetectable serum concentrations are verified (up to 2 years!) -Two tests performed at least 2 weeks apart Elimination procedure: Cholestryamine 8g TID for 11 days (also can be used for males) -repeat PRN until undetectable serum concentration reached Breastfeeding: contraindicated

You want to start your patient on MTX, what counseling should you provide?

Counseling: -avoid alcohol, especially on day of MTX administration -contraception counseling -folic acid required every day CBC, LFT's EVERY 3 MONTHS CXR at baseline due to possible pulmonary fibrosis

A (T1/T2) _____ weighted MRI shows the best pathology.

T2-weighted Black (low intensity) -Fat if fat saturated image -White matter Gray (intermittent intensity) -Muscle -Gray matter White (high intensity) -Fluid (urine, CSF) -Fat if not fat saturated image CSF is bright on T2!

T or F: TNF-alpha inhibitors must be refrigerated, which may cause issues for those who travel or do not have continued access to a refrigerator.

TRUE

A 75-year-old woman presents to the emergency department with complaints of severe headache, jaw claudication, and visual changes. Temporal artery biopsy results are positive for giant cell arteritis. Which of the following is the most appropriate therapy? A. Hydrocodone B. Methotrexate C. Naproxen D. Prednisone

D. Prednisone Temporal Arteritis (Giant Cell Arteritis) Patient will be a woman > 50 years old Monocular visual loss, unilateral headache, jaw claudication PE will show a tender temporal artery Labs will show ESR > 50 mm/hour Diagnosis is made by temporal artery biopsy Treatment is high-dose steroids ASAP Associated with polymyalgia rheumatica

A 25-year-old woman comes to the emergency department because of fatigue and widespread pain for the past 5 months. She says that she feels tired all the time and that her shoulders, back, and legs ache constantly. More recently, she has been feeling hopeless that these symptoms will not go away and it has been affecting her job performance. Her temperature is 37.5°C (99.5°F), pulse is 76/min, respirations are 12/min, and blood pressure is132/88 mmHg. Physical examination shows tenderness in her shoulders, back, and legs. There is no joint tenderness, crepitus, or deformity of her joints. Her skin and mucous membranes are moist and her thyroid gland is not palpable. Which of the following is the most appropriate pharmacotherapy? A. Clonidine B. Lorazepam C. Oxycodone D. Pregabalin E. Venlafaxine

D. Pregabalin This patient presents with a fairly classic presentation of fibromyalgia. Fibromyalgia is chronic (>3mo) and very widespread condition. Patients have nonarticular pain with characteristic tender points. Woman are more commonly affected than men. It most commonly occurs in patients between the age of 25-45. Patients often complain of symptoms such as widespread aching, stiffness, easy fatiguability, sleep disturbance, emotional stress, and tenderness with palpation of soft tissues with no specificity. Treatment of fibromyalgia is multimodal. Initial treatment can be conservative with proper education on fibromyalgia, physical therapy, and stress reduction. It should include physical therapy, counselling, an exercise program, and medication. There are several different medications that are of proven benefit in the treatment of fibromyalgia. Medications that can be used in the treatment of fibromyalgia include tricyclic antidepressants, anticonvulsants, and analgesics. Tricyclic antidepressants are a common initial treatment (i.e. amitriptyline). If patients do not tolerate a TCA, pregabalin (Lyrica) is another typical medication to try. Pregabalin is derived from GABA and is sold under the trade name "Lyrica." Its mechanism of action is unknown, but it may regulate calcium channel function. It is FDA approved for fibromyalgia.

An 11-year-old boy comes to the clinic with his mother because of pain in his right knee. He says that the pain started approximately a month ago and often wakes him up during the night. His mother also says that the pain is not relieved by warm compresses but does improve when she gives him aspirin. His temperature is 37.5°C (99.5°F), pulse is 60/min, respirations are 13/min, and blood pressure is 119/79 mmHg Physical examination shows some localized pain over the area of the proximal tibia. Thereis no palpable joint deformity, redness or swelling. Which of the following radiographic findings is the most likely explanation for this patient's symptoms? A. A 3-cm broad based excrescence projecting from metaphysis of upper tibia B. A 6-cm expansile mass in diaphysis of right lower femur with an periosteal reaction C. A 7-cm mass in the distal femoral epiphysis D. Ill-defined mass involving the femur metaphysis with elevation of periosteum E. Well defined 1-cm lucency surrounded by a thin rim of bony sclerosis in the proximal tibia

E. Well defined 1-cm lucency surrounded by a thin rim of bony sclerosis in the proximal tibia The question stem describes findings characteristic of osteoid osteoma, a benign bone tumor commonly found in children and young adults (age 10-20). An osteoid osteoma has woven bone centrally and a sclerotic rim. Pain disproportionate to the tumor size and response to aspirin are classic presentations of this tumor. The acute pain is mediated by release of prostaglandins. Therefore, aspirin is an effective analgesic to relieve pain in these patients. If such a lesion is larger than 2 cm, it is classified as an osteoblastoma. Osteoblastomas most commonly arise in vertebral posterior elements and are uncommon in the appendicular skeleton like osteoid osteomas. Radiographic imaging shows a well defined lucency surrounded by a thin rim of bony sclerosis. Treatment of osteoid osteomas consists of radio- frequency ablation or curettage.

_______________ is a good choice in mild disease or if other therapies are contraindicated. It is a useful combination agent, and is considered the cornerstone drug for patient's with Lupus (especially Lupus AND RA).

Hydroxychloroquine Time to benefit: 2-6 months ADR's: -Common: GI adverse events (N/V/D), patient counseling take with food or milk -Serious: retinal toxicity Monitoring: -CBC, LFTs -Ophthalmologic exam at baseline and every 3-6 mo during prolonged therapy Special populations: -safe for use during pregnancy -limited data for breastfeeding, probably safe

(IL-1/IL-6) ____________ receptor antagonists are used for patients with low response to TNF inhibitors. They have a black box warning for serious infection risk, and also have risks of hyperlipidemia, neutropenia, thrombocytopenia, hepatotoxicity, GI perforation, and cancer.

IL-6 Use is often limited by SE's. DO NOT use in those with hepatic impairments, pregnancy, or breastfeeding Do not initiate if: 1. ANX <2000 2. Platelets <100,000 3. ALT or AST >1.5x ULN 4. Active infections

____________ myositis responds poorly to steroids and immunosuppressive therapies.

Inclusion body

In (inclusion body myositis/polymyositis) ___________ the muscle biopsy will show ringed vacuoles and filamentous inclusions.

Inclusion body myositis In inclusion body myositis, there is involvement of distal muscles (foot extensors & finger flexors). Asymmetric weakness and atrophy is also seen.

What is the MOA of Febuxostat?

Inhibition of Xanthine Oxidase

Which of the following drugs works by preventing activation, degranulation, and migration of neutrophils? a. indomethacin b. prednisone c. allopurinol d. colchicine

d. colchicine

A 35-year-old woman comes to the office because of back pain for 12 years. She describes pain from her neck to tailbone. It is worse at night, sometimes even waking her from sleep. The pain subsides with movement and exercise. She also mentions having intermittent pain at the elbows, wrist, ankles and most recently, the knees. Physical exam shows restricted movements of the neck, the thoracic spine and her lumbar spine. Her ability to expand her chest was reduced, and she was tender at the sacroiliac joints. Which of the following is least likely to be found in her diagnostic work-up? A. Anti-CCP positive B. Elevated C-reactive protein C. Elevated erythrocyte sedimentation rate D. HLA-B27 positive E. Sacroiliitis

A. Anti-CCP positive Ankylosing spondylitis (AS) is a chronic inflammatory disease characterized by involvement of the spine and sacroiliac (SI) joints, peripheral joints, and entheses. AS typically develops in young adults, with a peak age of onset between 20 and 30 years, and has a slight preponderance for males. Inflammatory back pain is present in 70 to 80 percent of patients with AS. It typically exhibits at least four of the following five features: Age of onset < 40 years, insidious onset, improvement with exercise, worsening upon rest, and pain at night (and improvement upon rising). Other musculoskeletal findings include postural abnormalities, buttock pain, hip pain, peripheral arthritis, enthesitis, and dactylitis. Extra-articular co-morbidities include inflammatory bowel disease, acute anterior uveitis, psoriasis, cardiovascular disease, and pulmonary disease. This patient has ankylosing spondylitis, a seronegative arthropathy which as the name suggests would be negative for serologies such as anti-cyclic citrullinated peptide antibodies, which is a very specific test for rheumatoid arthritis. An elevated erythrocyte sedimentation rate (ESR), HLA-B27 positive, elevated C-reactive protein (CRP), and evidence of sacroilitis are all consistent with AS. Serum bone-specific alkaline phosphatase may be elevated in cases of severe disease, and serum levels of IgA are frequently elevated.

A 43-year-old woman presents to her primary care physician with complaints of bilateral wrist and hand pain for the last two months. Her symptoms are the worst when she wakes up and improve throughout the day or when she takes ibuprofen. She denies recent injury, recent illnesses, or rash. Given her presentation, you suspect rheumatoid arthritis. Which of the following combinations provides the most specific diagnostic findings for this condition? A. Anti-citrullinated peptide/protein antibody and rheumatoid factor B. Antinuclear antibody and rheumatoid factor C. Antinuclear antibody and X-ray D. C-reactive protein and erythrocyte sedimentation rate

A. Anti-citrullinated peptide/protein antibody and rheumatoid factor Which joints in the hand are most commonly affected in patients with rheumatoid arthritis? Answer: The metacarpophalangeal (MCP) joints. Patient with RA should be treated symptomatically with nonsteroidal anti-inflammatory drugs (NSAIDs) and started early on disease-modifying antirheumatic drugs (DMARDs) to prevent joint injury and disability, after testing for tuberculosis, hepatitis B and hepatitis C, as patients with active hepatitis may require antiviral or immunosuppressive therapy before starting a DMARD. The first-line DMARD is methotrexate, which can inhibit folic acid metabolism. Thus, patients on methotrexate should concurrently be started on daily folic acid supplementation. Methotrexate is also known to cause bone marrow suppression, liver and lung toxicities, so management on methotrexate will also necessitate close monitoring for these toxicities. Additionally, there are multiple comorbidities associated with RA and medications like DMARDs used to treat RA including: cardiovascular disease, pulmonary disease, and malignancy. Patients under the care of a rheumatologist to monitor disease progression and drug toxicities and respond to disease flares, have better outcomes such as decreased joint injury and improved functional capacity, than those followed by other clinicians.

A 16-year-old girl comes to the clinic with a 9-month history of persistent fevers, malaise, and headache. Recently, she has noticed that her urine is occasionally dark or brown in color. She also notes new onset shortness of breath when exercising. She denies any illicit or prescription drug use. Her temperature is 38.3°C (101.0°F), pulse is 70/min, respirations are 16/min, and blood pressure is 110/76 mm Hg. Physical examination shows a faint erythematous rash over the upper cheeks and the bridge of the nose. Microscopy of a urine sample collected today is notable for the presence of several RBC casts. Which of the following is the most appropriate next diagnostic test? A. Anti-dsDNA and anti-Smith serum antinuclear antibodies B. Anti-histone serum antinuclear antibodies and p-ANCA C. Lung biopsy D. Magnetic resonance imaging of the head E. Prothrombin and activated partial thromboplastin times

A. Anti-dsDNA and anti-Smith serum antinuclear antibodies This patient likely has systemic lupus erythematosus (SLE), a nebulous collection of functions resulting from an autoimmune response to nuclear antigens. Specific antinuclear antibodies such as anti-dsDNA and anti-Smith antibodies are typically elevated in SLE; elevations of those two autoantibodies as well as a positive renal biopsy are sufficient to diagnose SLE. Other antibodies that are characteristically seen are anti-Ro/SSA and anti-La/SSB. The characteristic malar rash is absent in about 2/3 of patients presenting with SLE. The treatment for this disorder includes hydroxychloroquine and NSAIDs for more mild cases; for more intense cases, immunomodulators such as steroids, mycophenolate mofetil, azathioprine, methotrexate, or a combination of the above. Because of the severe side effect profiles of steroids & immunosuppressants, conservative therapy with hydroxychloroquine and NSAIDs is generally attempted first.

A 25-year-old woman presents to the office with a six-month history of diffuse joint pain, facial rash, hair loss, and fatigue. On physical examination, she has mild swelling of her hands without synovitis. She has patchy alopecia and erythema over her nose and cheeks. Laboratory testing reveals mild anemia and a positive antinuclear antibody with a titer of 1:160. Antibodies to double-stranded deoxyribonucleic acid are also present. Based on this patient's probable diagnosis, what is appropriate advice to give her about managing her condition? A. Avoid sun exposure B. Completely abstain from alcohol consumption C. Influenza vaccination is contraindicated D. Minimize physical activity

A. Avoid sun exposure Systemic Lupus Erythematosus (SLE) Patient often a woman Complaining of fever, lymphadenopathy, weight loss, general malaise, or arthritis PE will show malar rash ("butterfly rash") Labs will show anti-nuclear antibodies (ANA), anti-dsDNA antibodies, anti-Smith antibodies, anti-histone antibodies Treatment is NSAIDs, steroids, immunosuppressants, hydroxychloroquine Drug-induced lupus: Hydralazine, INH, Procainamide, Phenytoin, Sulfonamides (HIPPS) Best contraception - LNg-IUD or POP Diagnosis of systemic lupus erythematosus is made by a combination of clinical suspicion and measurement of antinuclear antibodies. If the antinuclear antibody titer is elevated, tests should be done for other autoantibodies. Anti-double-stranded deoxyribonucleic acid and anti-smooth muscle antibodies are specific for systemic lupus erythematosus. The erythrocyte sedimentation rate and C-reactive protein are usually elevated and may be used to monitor disease activity. Treatment involves disease management since it is a chronic disease. Hydroxychloroquine or chloroquine is typically used as a first-line treatment. Glucocorticoids and other immunosuppressive medications, such as azathioprine or methotrexate, may be indicated for severe disease or flares of disease. Lifestyle modifications, especially avoiding sun exposure, are helpful in managing symptoms. In addition, smoking cessation, moderate exercise, and a balanced diet are recommended. Some medications, such as sulfonamide-containing antibiotics, can exacerbate systemic lupus erythematosus and should be avoided. Pregnancy should be avoided while the disease is active as there is an increased risk of miscarriage, pregnancy complications, and worsening of the disease.

What physical exam finding in chronic rheumatoid arthritis is described as a position of the fingers in which the proximal interphalangeal joint is in permanent flexion and the distal interphalangeal joint is in permanent hyperextension? A. Boutonnière deformity B. Metacarpophalangeal deviation C. Swan neck deformity D. Z deformity

A. Boutonnière deformity Rheumatoid Arthritis More common in women and those aged 40-50 years Morning stiffness lasting > 30 minutes PE will show symmetrical soft, red, tender swelling in joints - MCP, PIPBilateral ulnar deviation at MCP, boutonniere deformity, and swan-neck deformity Labs will show positive RF, anti-cyclic citrullinated peptide antibodies Most commonly caused by autoimmune destruction of synovial joints Treatment is DMARDs Swan neck deformity (C) is another common physical exam finding in chronic rheumatoid arthritis. It involves proximal interphalangeal hyperextension and distal interphalangeal joint flexion. The curvature of this joint deformity resembles the curvature of a swan's neck.

A 70-year-old woman presents to the office with right knee pain for one week, which has been getting worse. On physical examination, she has an effusion in the knee, it is warm to touch, and she has pain with range of motion. Aspiration of synovial fluid reveals cloudy, yellow, nonpurulent fluid with a white blood cell count of 20,000 cells per cubic millimeter and rhomboid-shaped weakly birefringent crystals. Which of the following findings is most likely to be present on X-rays of her knee? A. Calcification of the menisci B. Ossification of the patellar tendon C. Osteopenia and marginal erosions D. Periarticular soft tissue opacities and "punched-out" erosions

A. Calcification of the menisci Pseudogout Complaining of monoarticular arthritis Labs will show rhomboid-shaped crystals, weakly positive birefringence Most commonly caused by calcium pyrophosphate crystals Management includes intra-articular steroid injection (2 or less joints), NSAIDs, colchicine, and systemic corticosteroids

Which of the following is the first line biologic therapy for most patients? A. TNF-alpha inhibitors B. Abatacept C. IL-6 receptor antagonists D. Rituximab

A. TNF-alpha inhibitors MOA: binds tumor necrosis factor alpha, thereby interfereing with binding to TNF-alpha receptor sites and subsequent cytokine-driven inflammatory processes -dec. signs and symptoms of RA-inhibits progression of structural damage in rheumatoid arthritis

A 45-year-old man presents to the emergency department with a spine injury after a motorcycle accident. He is alert and oriented and complains of severe pain in his lower back and both legs. On physical examination, he has decreased sensation over the perineum, weakness of plantar flexion and absent ankle jerk reflexes bilaterally, and fullness and tenderness in the suprapubic area. Which of the following is the most likely diagnosis? A. Cauda equina syndrome B. Central cord syndrome C. Disc herniation at the L5-S1 level D. Spinal stenosis

A. Cauda equina syndrome The cauda equina (Latin for "horse's tail") is the bundle of nerve roots below the level of the spinal cord. The spinal cord usually ends at the level of the L1 or L2 vertebrae but can end anywhere between T12 and L3. Cauda equina syndrome is compression of the nerve roots below the distal end of the spinal cord. Patients present with symptoms of lower back and leg pain, often severe; weakness and absent reflexes in the lower extremities; loss of sensation in a dermatomal distribution; saddle anesthesia; and urinary incontinence due to urinary retention and overflow incontinence. Physical examination shows decreased reflexes, sensory deficits, and decreased rectal tone (no "anal wink"). Magnetic resonance imaging or CT myelogram are indicated immediately, as cauda equina syndrome is a surgical emergency. Surgical decompression of the nerves should be performed within 24 to 48 hours. Studies have shown better return of bowel and bladder function and motor and sensory function when surgery is done within 48 hours of symptom onset.

A 33-year-old man comes to the office because of sore knees for the past 3 days. He says that he is also experiencing painful urination and painful, red, itchy eyes. He was treated for an infection 3 weeks ago. He has an uncle with ankylosing spondylitis. Examination shows bilateral swollen knees, with effusions. There is also bilateral injection of the conjunctivae. Which of the following microbes is the most likely cause of this patient's condition? A. Chlamydia trachomatis B. Clostridium botulinum C. Neisseria gonorrhoeae D. Rickettsia prowazekii E. Treponema pallidum

A. Chlamydia trachomatis Chlamydia trachomatis is the most common triggering infection for reactive arthritis, which is an autoimmune condition that develops because of cross-reactivity. Reactive arthritis is characterized by nongonococcal urethritis, asymmetric oligoarthritis, and conjunctivitis.

A 55-year-old man comes to the clinic because of chronic muscle weakness for 6 months. He is taking atorvastatin for treatment of hyperlipidemia. He had a history of gastrectomy because of gastric adenocarcinoma 2 years ago. Physical exam shows 3/5 strength of pelvic and shoulder muscles, 5/5 strength of distal muscles, and normal sensation. Skin examination shows bilateral hand rash shown in the image below..Which of the following is the most likely diagnosis? A. Dermatomyositis B. Drug side effect C. Eaton lambert syndrome D. Inclusion body myositis E. Polymyositis

A. Dermatomyositis Dermatomyositis is an idiopathic inflammatory myopathy characterized by skin findings, systemic manifestations, and proximal muscle weakness. Heliotrope rash and Gottron papules are pathognomonic for dermatomyositis. Anti-M-2 antibodies is the most specific MSA for dermatomyositis.

A 43-year-old man with a history of back pain is found to have a herniated disc of the lumbar spine at the level of L5. Physical exam reveals 4-5 strength and shows diminished sensation. Which of the following best describes the location of diminished sensation in this patient? A. Dorsal foot B. Lateral foot C. Medial ankle D. Posterolateral thigh

A. Dorsal foot Herniated Disk Pulposus Patient presents with abrupt onset of pain that is severe and exaggerated by sitting, walking, standing, or coughing Pain radiates from the buttock down the posterior or posterolateral leg to the ankle or foot PE will show positive straight leg raise, tenderness in the sciatic notch, and limited ROM Diagnosis is made by MRI Most commonly caused by herniation of the nucleus pulposus into the lumbar spinal canal Most common location L4 to L5 or L5 to S1 The diagnosis of herniated nucleus pulposus is best confirmed with MRI, which delineates the level and severity of herniation. First-line treatment includes nonsteroidal anti-inflammatory drugs and physical therapy. Epidural injections and surgery are options for patients who do not respond to conservative methods.

A 37-year-old woman comes to the office because of fragile nails and fingertip ulcerations. She says that prior to the lesions her fingertips and toes would undergo strange color changes with associated numbness. However, she did not think that this was significant and she attributed it to the recent cold weather. She has a history of gastroesophageal reflux disease (GERD), hypertension, and asthma. She drinks alcohol occasionally but does not smoke. Her temperature is 37.5°C (99.5°F), pulse is 70/min, respirations are 14/min, and blood pressure is 130/79 mmHg. Physical examination reveals digital ulcerations and patched of lighter-colored skin. Cardiac and respiratory examinations are normal. Which of the following is a symptom that can be associated with her disease? A. Dysphagia and heartburn B. Hypercoagulable state C. Thickened patches of skin around the genital area D. Vesicular skin lesions E. Acanthosis nigricans

A. Dysphagia and heartburn This patient is suffering from localized systemic sclerosis. The clinical symptoms such as dysphagia, heartburn, digital ulcerations, the presence of Raynaud's phenomenon, and pruritis are common in scleroderma (systemic sclerosis). Scleroderma is a multisystem, autoimmune disease of no known cause, resulting in vascular abnormalities, fibrosis, and characteristic symmetrical skin thickening. GI involvement of scleroderma includes heartburn and dysphagia (of both solids and liquids) because of esophageal dysmotility from smooth muscle conversion to collagen. Other manifestations include dyspnea, due to interstitial fibrosis, renal disease, tightened facial features and polyarthralgia. A syndrome that is associated with localized systemic sclerosis is CREST syndrome. This acronym refers to calcinosis, Raynaud phenomenon, esophageal dysmotility, sclerodactyly, and telangiectasia.

A 49 year-old woman comes to the clinic because of tightening of the skin on her hands and face. She has a history of hypertension and asthma. She is currently taking amlodipine, losartan, and salbutamol. She does not smoke or drink alcohol. Her temperature is 37.5°C (99.5°F), pulse is 70/min, respirations are 14/min, and blood pressure is 145/87mmHg. Physical examination shows non-pitting edema and some ulceration on her hands. Her abdomen is not distended. Bowel sounds are hypoactive. She does not have stridor but there is some diffuse bilateral wheezing on auscultation with good air entry bilaterally. Cardiac examination is normal. Which of the following organs is most likely to be affected by the patients disease? A. Esophagus B. Heart C. Liver D. Spinal cord E. Spleen

A. Esophagus The patient is suffering from localized system sclerosis, or scleroderma. Systemic sclerosis is a non- inflammatory autoimmune disorder characterized by widespread small vessel vasculopathy and fibrosis.This process leads to atrophy and fibrosis of tissues. Intimal proliferation and degeneration of vessel media leads to progressive obliteration of vessel lumen. Patients often develop hypertension as evident in this patient. Limited scleroderma affects the skin of the face, hands, and neck. The skin on other areas of the body is not affected. Systemic sclerosis can affect much of the GI tract, with the esophagus being the most common portion affected, frequently resulting in dysphagia. Findings on lung exam may represent an early finding of interstitial pulmonary fibrosis, a complication of systemic sclerosis that can be fatal. It affects women more commonly than men and often develops in patients between the age of 40-50. An anti-neutrophil cytoplasmic antibody (ANCA) should be done to aid in the diagnosis. The anti-centromere antibody is positive in patients with limited system sclerosis. Treatment varies depending on the symptoms that the patient is exhibiting.

A 55-year-old woman presents with a gradual onset of a cough, dyspnea, and red eyes for 11 months. She has been treated multiple times for otitis media and sinusitis. She also reports ankle pain that has migrated to her knee. Physical exam reveals saddle nose deformity, nasal septum crusting, anterior uveitis, and otitis media. Laboratory tests reveal a positive antineutrophil cytoplasmic antibody directed towards proteinase-3. Anticyclic citrullinated peptide antibody and rheumatoid factor are absent. Tissue biopsy shows granulomatous inflammation and geographic necrosis. Which of the following is the most likely diagnosis? A. Granulomatosis with polyangiitis B. Microscopic polyangiitis C. Refractory sinusitis D. Rheumatoid arthritis

A. Granulomatosis with polyangiitis Granulomatosis With Polyangiitis Formerly known as Wegener granulomatosis Positive ANCA Thorough history/physical Affects kidneys and respiratory tract Diagnosis by biopsy Consider chest imaging Treat with steroids and immunosuppressive agents

A 19-year-old man comes to the emergency room because of difficulty breathing for 6 hours. He reports that earlier that day, he was doing laundry and suddenly became short of breath. He also reports back pain for a year that is worst after waking up in the morning. He reports that his morning stiffness typically lasts for approximately an hour. Over the past few months, he has noticed pain in his hips intermittently but did not have insurance to see a doctor so he ignored it. Chest and spine X-rays are obtained. Which of the following is associated with the most likely diagnosis? A. HLA-B27 positivity B. HLA-DR3 positivity C. Positive ANA D. Positive cyclic citrullinated peptide E. Positive rheumatoid factor

A. HLA-B27 positivity Ankylosing spondylitis should be suspected in a young male with inflammatory back pain - pain that is worse in the morning and is accompanied by stiffness for more than 30-60 minutes. Radiographs commonly show a "bamboo spine" secondary to the fusion of the vertebral bodies. The diagnosis is associated with HLA-B27 positivity

A 6-year-old boy is brought to the clinician's office by his parent because of a 1-day history of rash, nausea, and stomach pain. The parent reports the patient had a throat infection several weeks ago which resolved spontaneously. A few days later, he developed a hip pain that subsided and was followed by right knee and ankle pain. At the same time, he developed a rash on his legs, buttocks, and forearms. Immunizations are up to date. Vitals are within normal limits. Physical examination reveals a rash, as shown in the image below. Musculoskeletal examination reveals a swollen right knee without effusion, erythema, or warmth. Scrotal swelling and sensitivity are noted on the left side. Urinalysis shows microscopic hematuria with proteinuria. Serum electrolytes, including creatinine, are within normal limits. Retrieved from: Wikimedia CommonsWhich of the following is the most likely diagnosis? A. IgA vasculitis B. Childhood-onset systemic lupus erythematosus C. Juvenile idiopathic arthritis D. Rheumatic fever E. Mumps infection

A. IgA vasculitis IgA vasculitis (Henoch-Schönlein purpura) is characterized by the classic clinical triad of palpable purpura petechiae, migratory arthritis, and abdominal pain. It usually occurs after a viral infection or streptococcal infection.

A 65-year-old man comes to the emergency department because of severe pain in his left ankle for the past eight hours. He denies traumatic injury. He has had two previous episodes of joint pain that were not as severe, once in his right ankle and once in his right first toe. He states that he eats a high protein diet and drinks six to eight beers daily. His temperature is 37.1°C (98.8°F), pulse is 90/min, respirations are 16/min, and blood pressure is 136/72 mm Hg. Physical examination shows the ankle is swollen, red, warm, and extremely tender to the touch. The rest of the physical examination is benign. Which of the following additional findings is most likely associated with this condition? A. Insulin resistance or metabolic syndrome B. Prior ankle fracture C. Progressive hearing loss over the past year D. Self-mutilation by biting his fingers E. Unprotected sex with multiple partners

A. Insulin resistance or metabolic syndrome A history of insulin dependence and metabolic syndrome is a risk factor for gout. Gout was known as a disease of the wealthy throughout history. This social class often had a diet high in seafood, red meat, and alcohol - all of which increase uric acid levels. In acute gout attacks, the concentration of uric acid in the blood is high enough that it precipitates out and forms crystals, which accumulate in joint spaces and cause severe stabbing pain. Pain in the big toe is a classic presentation of gout and is referred to as podagra.

A 55-year-old woman that is post-menopausal comes to the clinic because of persistent right knee pain. She reports stiffness in the morning of the right knee more than the left with resolution after about 30 minutes. She also states that the pain is worst at night just before bed. Her temperature is 37.1°C (98.8°F), pulse is 66/min, respirations are 14/min, and blood pressure is 128/78 mm Hg. A radiograph is obtained of both knees and shows osteophyte formation, asymmetric joint space narrowing, and sclerosis of the right knee with only asymmetric joint space narrowing of the left knee. Which of the following is most likely degraded initially in the pathogenesis of this condition? A. Proteoglycans B. Mineralized bone C. Non-collagenous proteins D. Type I collagen E. Type II collagen

A. Proteoglycans Osteoarthritis is characterized by clinical and radiographic findings. Proteoglycans protect collagen from proteolytic degradation and is responsible for much of the resiliency of cartilage.

Which of the following drugs was issued a safety alert due to increases in heart related deaths? a. allopurinol b. pegloticase c. prednisone d. febuxostat

d. febuxostat Other SE's: arthralgia, abn LFT's MOA: prevent hypoxanthine from being made into uric acid

Which of the following tests is NOT routinely ordered for a patient on methotrexate? a. CBC b. LFT c. CXR d. hep B/C screening

d. hep B/C screening

(Abatacept/Rituximab) _____________ is a selective T-cell costimulation blocker that is often used in combination with MTX and requires hepatitis and TB monitoring.

Abatacept (Non-TNF inhibitor) Side effects: -URI -Nausea -Infection! -caution with COPD -> higher freq. of COPD related SE Monitor hepatitis and TB DDI: -biologics -leflunomide

NSAIDs tend to lessen pain in osteoid osteoma's because they inhibit prostaglandin release. These bone tumors are (malignant/benign) ____________.

Benign -Sxs: aching, night pain -Pain with prostaglandin release from nidus that is relieved with nsaids and not assoc with activity PE: LE Lesion, limp, swelling point tenderness Not typically able to identify on MRI

Ankylosing spondylitis is characterized by symmetric fusion of spine and sacroiliac joints, which may be accompanied by uveitis and ____________________.

Aortic regurgitiation

Which of the following medications is recommended prophylactically to decrease the frequency of attacks of gouty arthritis? A. Acetaminophen B. Allopurinol C. Ibuprofen D. Prednisone

B. Allopurinol Gout Patient will be a middle-aged man Acute onset of pain in the first MTP (podagra) Labs will show needle-shaped crystal with negative birefringence Most commonly caused by uric acid crystals Treatment Acute: NSAIDs, steroids, colchicine Chronic: allopurinol (first line) or probenecid Can be triggered by loop and thiazide diuretics

Which of the following is a first-line pharmacologic treatment for fibromyalgia? A. Acetaminophen B. Amitriptyline C. Exercise D. Ibuprofen

B. Amitriptyline Fibromyalgia Patient will be a woman Widespread musculoskeletal pain for > 3 months, nonrestorative sleep, and generalized fatigue PE will show tenderness at > 11 of 18 anatomic sites (trigger points) Labs will be normal Diagnosis is made clinically Treatment is education, antidepressants, avoid opioids Question: True/False: Glucocorticoids have shown efficacy in the treatment of fibromyalgia. Answer: False, trials have shown that glucocorticoids, such as prednisone, are no better than a placebo.

A 55-year-old woman presents to the emergency department with increasing pain in both legs for the past three days. She came in this morning because she was incontinent of urine. On physical examination, she has no tenderness to palpation of the back or legs. She has absent ankle jerk reflexes, weakness of plantar flexion, decreased sensation over the perineum and perianal area, and fullness and tenderness over the suprapubic area. X-rays of the spine show mild degenerative changes. Which of the following is the most likely diagnosis? A. Anterior cord syndrome B. Cauda equina syndrome C. Central cord syndrome D. Lumbar spinal stenosis

B. Cauda equina syndrome Cauda Equina Syndrome Patient will be someone with a history of trauma, malignancy, epidural abscess, or hematoma Acute onset of lower back pain with weakness and numbness PE will show urinary retention, saddle anesthesia, decreased rectal tone Diagnosis is made by MRI or CT myelogram Most commonly caused by a herniated disc Treatment is operative decompression

A 3-year-old child is brought to the emergency department by his mother because of a 6-day high fever. The fever has not responded to treatment with paracetamol. His temperature is 39°C (102.2°F). Examination shows an irritable appearing child with cracked lips and peeling skin on his fingertips. There is also injected conjunctiva without discharge, a swollen tongue, swollen cervical lymph nodes, and a maculopapular rash on his trunk. Which of the following complications is this child most likely at risk of if prompt treatment is not initiated? A. Acute renal failure B. Coronary artery aneurysm C. Monocular blindness D. Reye syndrome

B. Coronary artery aneurysm Kawasaki disease is a medium-vessel vasculitis seen in children under 5 years old. Diagnostic criteria include at least 5 days of fever, erythema of oral cavity or cracked lips, a rash on the trunk, erythematous swelling or skin peeling on hands and feet, nonsuppurative conjunctivitis and swollen lymph nodes in the neck. Coronary artery aneurysms occur in 20-25% of untreated children, usually detected after around 10 days of fever, although they may take several weeks to develop. In developed nations, it appears to have replaced acute rheumatic fever as the most common cause of acquired heart disease in children. Death can occur due to myocardial infarction after thrombosis or rupture of a large coronary aneurysm. Early treatment with intravenous immunoglobulin (IVIG) and aspirin is important to avoid this complication. Kawasaki disease is one of the few cases where it is acceptable to treat children with aspirin (generally aspirin is avoided in children because of the risk of developing Reye's syndrome). Other complications of Kawasaki disease may include valvular insufficiencies, aneurysms in other locations, intestinal obstruction or ischemia, and ophthalmologic changes including uveitis, amaurosis, and iritis.

A 67-year-old man comes to the clinic because of low back and buttocks pain for 3 weeks. He has no history of trauma, lower limb weakness, stool, or urine incontinence. He is a retired construction worker. He has type II diabetes and receives metformin twice per day. Physical examination shows lower back tenderness, decreased sensation in the foot bilaterally, mild weakness of the pedal pulse bilaterally, and normal muscle tone. Straight leg test is negative. Lower back X-ray shows no abnormalities. Lower back MRI shows decreased signal intensity of lumbar inter-vertebral discs on T1-weighted images. Which of the following is the most likely cause of the patient's symptoms? A. Ankylosing spondylitis B. Degenerative disc disease C. Lumbar disc prolapse D. Diabetic peripheral neuropathy E. Peripheral vascular disease

B. Degenerative disc disease Degenerative disc disease (DDD) is a multi-factorial process. DDD may be asymptomatic or patients may have back pain. MRI is the most specific test for diagnosing DDD. It shows decreased signal intensity of the inter-vertebral discs on T1-weighted images.

A 55-year-old woman comes to the emergency department with a worsening "skin rash." Her past medical history is noncontributory. This painful and pruitic red rash started 2 weeks ago and is most prominent on her elbows and cheeks. She also says her muscles feel weak when climbing stairs, trying to stand from chairs, and when combing her hair. Laboratory studies show an elevated creatinine kinase and elevated transaminases. Which of the following is the most likely diagnosis? A. Allergic rhinoconjunctivitis B. Dermatomyositis C. Drug rash D. Lupus E. Psoriasis

B. Dermatomyositis This patient has dermatomyositis caused by an inflammatory process in the muscles and skin. Patients can also present with Gottron papules (scaly, calcified nodules on the knuckles). Hallmarks include heliotrope rash (pictured above), and the "shawl sign." Suggestive lab findings include elevated creatinine kinase and lactate dehydrogenase (muscle damage). Multiple myositis-specific antibodies may also be present, but are positive in only about 30% of patients. The initial treatment of dermatomyositis is corticosteroids. Standard treatment is steroid medication and a small dose of chemotherapy. Dermatomyositis is often a paraneoplastic phenomenon, meaning it is a possible sequelae of cancer not necessarily due to presence of cancer cells. Therefore, investigation into cancer should be considered. Removal of underlying cancer, if applicable and possible, often results in remission of dermatomyositis.

A 75-year-old man presents to the emergency department with severe right knee pain that began yesterday. Examination shows a large effusion in the knee, warmth, erythema, diffuse tenderness to palpation, and pain with range of motion. Laboratory tests show a normal white blood cell count, normal uric acid level, and erythrocyte sedimentation rate of 50 mm/hr. Synovial fluid analysis yields a white blood cell count of 50,000 cells per cubic millimeter and needle-shaped, negatively birefringent crystals. Which of the following is the most likely diagnosis? A. Calcium pyrophosphate deposition disease B. Gout C. Reactive arthritis D. Septic arthritis

B. Gout Gout Patient will be a middle-aged man Acute onset of pain in the first MTP (podagra) Labs will show needle-shaped crystal with negative birefringence Most commonly caused by uric acid crystals Treatment Acute: NSAIDs, steroids, colchicine Chronic: allopurinol (first line) or probenecid Can be triggered by loop and thiazide diuretics

A 32-year-old woman presents with low back pain for the last two days. She states that it began gradually after carrying several large boxes upstairs. She has not tried anything to alleviate the pain. She denies paresthesias, numbness, and weakness. She denies any radiation of the pain into the lower extremities. Physical exam shows limited range of motion of the lumbar spine in flexion at endpoints, tenderness in the right paraspinal musculature without any midline tenderness, and normal heel and toe walking. Which of the following treatment plans would be most appropriate to implement at this time? A. Bed rest B. Heat therapy C. Massage therapy D. Physical therapy

B. Heat therapy Acute low back pain is common and associated with the following risk factors: sedentary job, female gender, smoking, obesity, advanced age, and depression. The majority of patients who present with back pain are diagnosed with nonspecific back pain. This is defined as back pain in the absence of a pathologic cause and is often associated with musculoskeletal strain. Serious etiologies must be ruled out with a detailed history and physical examination. History should include questions about changes in sensation, numbness, or weakness. Questions about fever, night sweats, or unintentional weight loss can help differentiate malignancy as the cause. Questions about intravenous drug use or recent infection can help rule out epidural abscess. Physical examination should focus on ruling out causes of back pain that warrant further diagnostics or imaging. Neurologic exam should include reflexes, strength, gait, and sensation of the lower extremities. Palpation of the midline that reveals tenderness may be a sign of vertebral metastases, compression fracture, or spinal infection. The straight leg raise will elicit pain associated with lumbar radiculopathy. Treatment options include nonpharmacologic and pharmacologic options. If patients wish to try nonpharmacologic options, heat therapy has been shown in studies to reduce muscle spasm. Other therapies including massage, acupuncture, and self-care exercises have not been shown to be as efficacious, however, patients may find some relief in these modalities. First-line pharmacologic therapy includes nonsteroidal anti-inflammatory drugs (NSAIDs). Acetaminophen has not been shown to be as helpful in alleviating pain. Second-line treatment includes a nonbenzodiazepine muscle relaxant. Prognosis is good with most cases resolving within seven weeks of onset.

Which of the following medications is most appropriate to treat an acute lumbar strain without radiculopathy? A. Diazepam 10 milligrams orally every six hours as needed for seven to 10 days B. Ibuprofen 600 milligrams orally every eight hours as needed for seven to 10 days C. Methylprednisolone beginning at 24 milligrams orally daily and tapering over six days D. Oxycodone 5 milligrams orally every four hours as needed for seven to 10 days

B. Ibuprofen 600 milligrams orally every eight hours as needed for seven to 10 days Acute lower back pain without neurologic symptoms, often called mechanical lower back pain, is a common presenting complaint. Most of the time, a specific cause cannot be identified, but injury to muscles and ligaments can occur with an acute strain. The patient may report a specific event, such as bending, twisting, or lifting, or there may be no history of injury. The examiner should ask about radicular symptoms, such as leg pain, paresthesias, and weakness; bowel or bladder dysfunction; and constitutional symptoms. Physical examination should focus on inspection, palpation for tenderness, range of motion of the spine and hips, and a focused neurologic examination. Imaging and laboratory tests are rarely necessary for isolated lower back pain without neurologic findings, such as decreased sensation, motor weakness, or loss of deep tendon reflexes. Most lower back pain will resolve within four weeks, so symptomatic treatment is indicated. However, some patients will develop chronic pain lasting more than three months, so follow up is recommended. Risk factors for the development of chronic back pain include smoking, obesity, physically or psychologically strenuous work, sedentary work, Workers' Compensation insurance, and psychologic factors such as somatization disorder, anxiety, and depression. Treatment for acute pain includes modalities such as heat and possibly massage, acupuncture, or spinal manipulation. It is important to counsel patients to avoid prolonged bed rest and to begin gradual exercise and resume activities as pain allows. Physical therapy may be helpful to prevent recurrent episodes of pain, especially in patients who may be at risk for the development of chronic back pain. Mild pain relievers, such as ibuprofen or other nonsteroidal anti-inflammatory medications, are preferred for short-term relief.

A 50-year-old woman comes to the office with the complaint of pain and stiffness in her shoulders. She also reports fatigue and weight loss. Physical examination shows 3/5 muscle strength in the proximal upper extremities bilaterally and 4/5 throughout the lower extremities bilaterally. She states that she can no longer climb the stairs at home. Which of the following laboratory findings do you expect be associated with this patient's condition? A. Decreased CK, normal aldolase, positive anti-Jo-1 antibodies B. Increased CK, increased aldolase, positive anti-Jo-1 antibodies C. Normal CK, increased aldolase, negative anti-Jo-1 antibodies D. Normal CK, normal aldolase, increased ESR E. Normal CK, normal aldolase, positive anti-Jo-1 antibodies

B. Increased CK, increased aldolase, positive anti-Jo-1 antibodies Polymyositis is a progressive, systemic connective tissue disorder characterized by immune mediated muscle inflammation primarily affecting striated muscle fibers. It usually affects patients between the ages of 50-70 and affects females more than males. It may present with bilateral symmetric progressive proximal muscle weakness. Patients with polymyositis may develop myocarditis and cardiac conduction defects and their condition may be associated with an underlying malignancy, especially breast and lung carcinomas. Lab analysis reveals an increased serum CK, increased aldolase and positive anti-Jo-1 antibodies. Dermatomyositis has a similar clinical presentation as polymyositis, but also involves the skin. Treatment consists of high dose corticosteroids. Azathioprine and/or methotrexate may be used as steroid sparing agents in patients intolerant to steroid therapy including patients with poorly controlled diabetes. Steroids have been shown to increase serum glucose levels which may be problematic in patients with poorly controlled diabetes.

Osteoid osteoma is a (benign/malignant) _______ bone tumor that is usually found in the spine or long bones.

Benign M>F, seen in young adults & adolescents. Path: nidus (atypical bone) enclosed with sclerotic reaction zone HPI: may cause aching, night pain -Release prostaglandins (cause pain) -Relieved w/ NSAIDs within 20-30 min -Pain is NOT related to activity Tx: Symptomatic (NSAIDs) -surgical removal vs. radiofrequency ablation (CT guided)

A 64 year-old man presents with 2 days of left knee pain. He is afebrile and has no signs of systemic infection. He denies trauma to the knee and states that the knee looks swollen and red and feels hot and extremely painful. A knee aspirate is done and polymorphonuclear cells within the joint space are found to contain crystals that are negatively birefringent under polarized light. If the patient is allergic to indomethacin, what is the mechanism of action for the medication you prescribe? A. Inhibition of xanthine oxidase and decrease in production of uric acid B. Inhibition of tubulin polymerization into microtubules by binding to tubulin C. Inhibition of prostaglandin synthesis by irreversibly binding COX-1 D. Stabilization of microtubules to prevent mitosis in rapidly-dividing cells E. Inhibition of production of the bacterial cell wall peptidogylcan cross links resulting in a bacteriocidal effect

B. Inhibition of tubulin polymerization into microtubules by binding to tubulin The patient is having an acute attack of gout. Gout is a crystal disease that is diagnosed when PMNs aspirated from the knee show intracellular crystals which are negatively birefringent under polarized light. For acute attacks, NSAIDs may be used for pain and colchicine is commonly used as well. The mechanism of action of colchicine is by binding to and inhibiting tubulin assembly in cells. This inhibits migration of neutrophils into the joint space and decreases the inflammatory response by decreasing mitotic divisions. Choice A is incorrect; this mechanism describes allopurinol which may be used for long term treatment in gout patients but should never be started or stopped during an acute attack because any change in uric acid metabolism may exacerbate the attack. Choice C is incorrect; this mechanism describes aspirin, not colchicine. Choice D is incorrect; this mechanism describes docetaxel which is used mainly in the treatment of breast and ovarian cancers. Choice E is incorrect; this mechanism describes penicillin which is not indicated in this synovial crystal disease. Antibiotics are indicated in the case of a septic joint.

A 32-year old man comes to the emergency department because of fever, worsening pain, and impaired range of motion in his right knee for 3 days. He states that he often has to clear small nails or glass splinters from the rough pavement of his construction site before kneeling to do work-related tasks. He has never had a prosthetic implant, there is no family history of rheumatoid arthritis or gout, and that he does not have unexplained skin lesions. Examination shows there is an obvious effusion in his right knee. It is visibly red and swollen, and warm and painful to touch. He refuses to bend it. His left knee is unaffected. Which of the following would be the most helpful test in establishing a diagnosis? A. Complete blood count and cultures B. Joint fluid analysis and culture C. Polarizing microscopy of synovial fluid for crystals D. Right knee plain film E. Right knee ultrasound scan

B. Joint fluid analysis and culture Septic arthritis, workup should involve a joint aspirate to confirm the infecting organism, which is often Staphylococcus aureus.

Which of the following is the most common site for development of an Ewing sarcoma tumor? A. Irregular bones B. Long bones C. Sesamoid bones D. Short bones

B. Long bones Ewing Sarcoma Patient presents with fever, weight loss, pain, and foot swelling that awakens them at night X-ray will show primarily lytic bone lesion with periosteal reaction (onion-skinning) Most common site is proximal femur Treatment includes chemotherapy, red blood cell, neutrophil and platelet support, and surgery. Mets commonly travel to lungs Possible petechiae and purpura due to thrombocytopenia 2nd most common pediatric malignant bone tumor - fertility counseling prior to starting treatment

A 55-year-old woman presents to the clinic with a 10-month history of extremely dry eyes that prevent her from wearing her contacts. In addition, she notes that four months ago she began noticing a "cotton mouth" sensation and difficulty swallowing foods. Which of the following most accurately describes the pathophysiology of the suspected diagnosis? A. Lymphocytic infiltrate in the endocrine glands B. Lymphocytic infiltrate in the exocrine glands C. Neutrophilic infiltrate in the endocrine glands D. Neutrophilic infiltrate in the exocrine glands

B. Lymphocytic infiltrate in the exocrine glands Lymphocytic infiltrate in the exocrine glands is characteristic of the pathophysiology of Sjögren syndrome. Sjögren syndrome is a systemic autoimmune disorder that causes dysfunction of the lacrimal and salivary glands. The condition is nine times more common in women and occurs most commonly between 40 and 60 years of age. Sjögren syndrome can occur in isolation, termed primary Sjögren syndrome, or in association with other rheumatologic diseases, termed secondary Sjögren syndrome. Patients with Sjögren syndrome present with keratoconjunctivitis sicca resulting from inadequate tear production caused by lymphocyte and plasma cell infiltrate of the lacrimal glands. Patients describe burning, itching, and a foreign body sensation in their eye, which may be manifested by the inability to wear contact lenses. They also complain of a "cotton mouth" sensation, which causes difficulty swallowing foods and may even cause difficulty speaking. Sjögren Syndrome Patient will be a woman 40 - 60-years-old Complaining of dry eyes (Xerophthalmia) and dry mouth (Xerostomia) Labs will show SSA (anti-Ro) or SSB (anti-La) Diagnosis is made by positive Schirmer test Most commonly caused by an autoimmune disorder Higher risk of lymphoma

A (benign/malignant) ________ primary bone tumor typically presents as well-defined with sclerotic margins.

Benign Suggestive of slow growth Malignant Malignant typically present with pain usually with palpable mass. Xray shows a permeative lesion w/ lytic destruction poorly defined margins which suggests rapid growth.

Which of the following diagnostic studies is most effective for evaluating tumor size in a patient with chondrosarcoma? A. Bone density scan B. Magnetic resonance imaging C. Plain radiograph D. Ultrasound

B. Magnetic resonance imaging The most common presenting symptom of chondrosarcoma is a deep, dull pain that occurs at night. Swelling may also be present at the site of the tumor. If the tumor is close to a neurovascular bundle, such as in the pelvis, neuropathy may occur. If it is close to a joint, range of motion and function may be affected. Patients may also present due to pathologic fracture of the affected bone. Workup for chondrosarcoma typically relies on the use of diagnostic imaging. Magnetic resonance imaging (MRI) or computed tomography is used to evaluate tumor size and local extent of the tumor. MRI can also help determine the extent of bone marrow and soft tissue involvement in the disease. MRI is also used to diagnose or confirm the return of chondrosarcoma at a surgically-treated site. Treatment involves surgery for all types of chondrosarcomas. Chemotherapy and radiation therapy play limited roles in the primary treatment of the disease,but may be used in select cases and for palliative care. Prognosis depends on the grade and stage of the tumor at initial diagnosis. Conventional central chondrosarcomas make up the majority of these tumors and affect the axial skeleton with the pelvis and ribs being the most common site.

Which of the following medications used to treat rheumatoid arthritis has been shown to alter the disease course and improve radiographic outcomes? A. Ibuprofen B. Methotrexate C. Naproxen D. Prednisone

B. Methotrexate Rheumatoid Arthritis More common in women and those aged 40-50 years Morning stiffness lasting > 30 minutes PE will show symmetrical soft, red, tender swelling in joints: MCP, PIPBilateral ulnar deviation at MCP, boutonnière deformity, and swan-neck deformity Labs will show positive RF, anti-cyclic citrullinated peptide antibodies Most commonly caused by autoimmune destruction of synovial joints Treatment is DMARDs

A 19-year old man comes to the office because of a progressive, painless mass over his right knee. His medical history is noncontributory, the patient denies a history of trauma, fever, or any other disease. When asking the patient about alleviating and exacerbating factors, he mentions that the pain is very intense whenever he decides to go exercise. Physical exam shows, a hard, immobile, and painless palpable mass, in addition to, muscle atrophy, and a limited range of motion of his right knee. His temperature is 36.1°C (96.9°F), pulse is 94/min, respirations are 19/min, and blood pressure is 110/80 mmHg. A conventional knee radiograph is obtained and shown below. This patient most likely has which of the following conditions? A. Osteoblastoma B. Osteochondroma C. Ewing's sarcoma D. Osteosarcoma E. Giant cell tumor

B. Osteochondroma Osteochondroma is the most common benign bone tumor and usually, occurs as a solitary lesion in the metaphyseal region of the long bones. This tumor is common in male adolescents and the hallmark clinical presentation is a progressive, painless growing mass.

A 35-year-old woman comes to the clinic for a health maintenance exam. She states she is currently feeling well. Current medications include a lipid-lowering agent and a corticosteroid. Her temperature is 37.1°C (98.8°F), pulse is 76/min, respirations are 18/min, and blood pressure is 132/84 mm Hg. Physical examination is unremarkable except for a malar rash, which she reports is exacerbated by sun exposure. For which of the following sequelae is this patient at increased risk? A. Osteoarthritis B. Osteonecrosis of the femoral head C. Paget's disease of the bone D. Slipped capital femoral epiphysis E. Spondylolisthesis

B. Osteonecrosis of the femoral head Systemic lupus erythematosus (SLE) places patients at increased risk for osteonecrosis. High-dose corticosteroid therapy for SLE adds to this risk. This patient has characteristic findings of systemic lupus erythematosus (SLE). Corticosteroids are used in the treatment of SLE as anti-inflammatory agents. Patients with SLE are at increased risk for osteonecrosis of the femoral head (image). Approximately 3-40% of SLE patients will develop osteonecrosis and chronic systemic corticosteroid can only add to this risk. It has also been found that SLE patients being treated for hyperlipidemia in combination have an increased risk for osteonecrosis. Osteonecrosis is often asymptomatic and is only treated upon complication. If bony collapse occurs, the only treatment option may be joint replacement. Osteonecrosis is generally prevented by not using high dose steroids in the treatment of SLE, if it can be avoided.

Which of the following is most likely to be found on physical examination of a 77-year-old woman with lumbar spinal stenosis? A. Increased pain with Valsalva maneuver B. Pain with extension of the spine C. Positive straight leg raise test D. Weakness with resisted knee extension

B. Pain with extension of the spine Spinal stenosis involves narrowing of the spinal canal and neural foramina, typically due to degenerative changes. As the intervertebral discs age, they become desiccated and lose height, causing bulging of the discs into the spinal canal and increased pressure on the facet joints. The joints become arthritic and enlarged, with osteophyte formation, which further narrows the spinal canal and neural foramina. A less common cause of spinal stenosis is a congenitally small spinal canal, for example in patients with achondroplasia. Patients present with lower back or buttock pain radiating to the legs, which is worse with walking or standing and relieved by bending forward or sitting. Patients often complain of leg pain that occurs with walking, known as neurogenic claudication. The pain is relieved by sitting down or leaning forward (the "shopping cart sign"). On physical examination, patients will typically have limited extension and pain with extension of the lumbar spine. Patients typically have a normal neurologic exam, and nerve tension signs are absent, however, patients with severe or longstanding stenosis may have weakness or loss of reflexes in the lower extremities. X-rays show degenerative changes of the intervertebral discs and facet joints. Magnetic resonance imaging shows hypertrophy of the facet joints and ligaments, desiccation of the intervertebral discs, decreased disc height, and narrowing of the neural foramina and spinal canal, which appears trefoil-shaped instead of round on axial images. Initial treatment is nonsurgical, with acetaminophen or nonsteroidal anti-inflammatory medications, physical therapy, and weight loss. Epidural glucocorticoid injections are often used but have not been shown to be better than local anesthetic injections. Surgery to decompress the spinal canal may be indicated. When is surgery indicated for spinal stenosis? Answer: For pain that has not improved after at least three to six months of nonsurgical treatment or for progressive neurologic deficits.

A 45-year-old man presents to your office with a complaint of worsening asthma symptoms. He tells you that he has had asthma since he was a child and is compliant with his daily inhaled glucocorticoid that has controlled his asthma until recently. He also reports recent sinus pressure. Labwork results show eosinophilia of more than 10% in a peripheral blood smear. To confirm the diagnosis of eosinophilic granulomatosis with polyangiitis, which of the following also needs to be present? A. Hypotension B. Paranasal sinusitis C. Vertigo D. Vomiting

B. Paranasal sinusitis Eosinophilic Granulomatosis Vasculitis Presents in phases Asthma, rhinosinusitis, eosinophilia ANCA Treat with steroids, cyclophosphamide, immunosuppressive agents

A 50-year-old man comes to the clinic due to fever, weight loss and weakness in his right foot and left hand for the past 2 months. During this period, he lost 4.5-kg (10-lb) without intention or changes in his diet. He has no jaw pain, change in vision, or headaches. He last visited his primary care physician more than 10 years ago, and his medical history is significant for IV drug use in his 20s. He no longer uses illicit drugs and does not smoke cigarettes. Temperature is 38.0°C (100.4° F), pulse is 80/min, and blood pressure is 145/90 mm Hg. Physical examination shows high-stepping gait when the patient lifts the right foot and decreased strength in the extensors of the left hand. There is no pain with palpation of the scalp. Cardiopulmonary exam shows no abnormalities. Skin examination shows purpura. Laboratory studies show the following: Laboratory value Result Hemoglobin 10 g/dL Hematocrit 31% Leukocyte count 15,000/mm3 Erythrocyte sedimentation rate 70 mm/h C-reactive protein concentrations 30 mg/dL (N=0.08-3.1) Serum Hepatitis B surface antigen Positive Perinuclear anti-neutrophil cytoplasmic antibodies Negative Urine Protein 2+ Erythrocytes 5-7/hpf Creatinine 2.5 mg/dL Which of the following is the most likely diagnosis? A. Eosinophilic granulomatosis with polyangiitis B. Polyarteritis nodosa C. Microscopic polyangiitis D. Temporal arteritis E. Thromboangiitis obliterans F. Granulomatosis with polyangiitis

B. Polyarteritis nodosa Polyarteritis nodosa is a systemic medium-vessel vasculitis affecting middle aged men that is often associated with hepatitis B infection.

A 40-year-old woman presents to the clinic with pain and morning stiffness in her fingers and toes. She describes the morning stiffness as lasting more than 30 minutes but says it improves throughout the day. On physical examination, you note a "sausage appearance" of her fingers and toes along with joint line tenderness and effusion of the distal interphalangeal joints. You also note onycholysis and nail pitting. There is an erythematous, thick plaque with silvery scale noted around her umbilicus. Laboratory results reveal an elevated erythrocyte sedimentation rate and uric acid levels, however, rheumatoid factor and anti-cyclic citrullinated peptide antibodies are absent. The synovial fluid examination does not show any monosodium urate crystals. Radiography of the fingers and toes shows a "pencil-in-cup" deformity. Which of the following is the most likely diagnosis? A. Gout B. Psoriatic arthritis C. Reactive arthritis D. Rheumatoid arthritis

B. Psoriatic arthritis Psoriatic Arthritis Patient with a history of psoriasis Dactylitis, various patterns of joint involvement CASPAR criteria (permit the diagnosis of PsA in spite of low rheumatoid factor positivity) Nail lesions Treat with DMARDs, biologic agents, PT/OT

A 35-year-old woman presents to the office complaining that over the past several months her fingertips have been visibly getting pale and numb, especially in cold weather. After they are warmed, her affected fingers become bright red with painful numbness and tingling throughout. She also notes that the skin on both hands has become dry and feels tight, especially on the palmar surfaces. A blood test is positive for antinuclear antibodies and anticentromere antibodies. Which of the following is the most likely diagnosis? A. Multiple sclerosis B. Scleroderma C. Sjogren's syndrome D. Systemic lupus erythematosus

B. Scleroderma Scleroderma Patient will be complaining of fatigue, stiff joints, pain PE will show thickening and hardening of the skin Limited cutaneous scleroderma causes CREST syndrome - Calcinosis of the skin, Raynaud phenomenon, Esophageal dysmotility, Sclerodactyly, and Telangiectasia Labs will show anti-topoisomerase I (anti-Scl-70) antibody (specific for diffuse disease) anti-centromere antibody (specific for limited disease) Most commonly caused by an autoimmune disease Treatment is targeted toward symptomatic relief, such as calcium channel blockers for severe Raynaud syndrome and antihypertensives, particularly angiotensin-converting enzyme inhibitors, for renal disease. Immunosuppressive agents may be given for significant pulmonary involvement as well as widespread multiorgan involvement. Prognosis tends to be worse in patients with diffuse scleroderma, in blacks, in the elderly, and in men. Lung disease in the form of either pulmonary fibrosis or pulmonary arterial hypertension is the most common cause of death. Other causes of death include heart failure or chronic kidney disease.

Which of the following TNF-alpha inhibitors must be given in combination with MTX? (2 answers) a. Adalimumab (humira) b. Etanercept (embrel) c. Golimumab (simponi) d. Infliximab (remicade)

C & D

A 65-year-old woman with a history of breast cancer presents complaining of a rash, fatigue, and difficulty climbing stairs for three months duration. Physical exam reveals heliotrope rash on her eyelids, erythema over her shoulders and back, and flat-topped violaceous papules over her knuckles. Which of the following lab findings is most specific for the suspected diagnosis? A. Anti-centromere antibody B. Anti-double-stranded DNA C. Anti-Jo-1-antibody D. Anti-Scl-70-antibody

C. Anti-Jo-1-antibody Dermatomyositis Patient will be a woman Insidious, painless, proximal muscle weakness (polymyositis) and a rash PE Malar rash Heliotrope rash Gottron papules Labs will show ↑ CK and aldolase Diagnosis is made by EMG, muscle biopsy Treatment is steroids Increased risk for malignancy (in adults)

A 35-year-old woman with a history of rheumatoid arthritis presents with persistent ocular and oral dryness. Physical exam shows xerostomia, keratoconjunctivitis sicca, dental caries, and parotid gland swelling. The Schirmer test is positive. Which of the following autoantibodies is most consistent with the diagnosis? A. Anti-double-stranded DNA B. Anti-Jo-1 C. Anti-SS-A D. Anti-topoisomerase-1

C. Anti-SS-A Sjögren Syndrome Patient will be a woman 40 - 60-years-old Complaining of dry eyes (Xerophthalmia) and dry mouth (Xerostomia) Labs will show SSA (anti-Ro) or SSB (anti-La) Diagnosis is made by positive Schirmer test Most commonly caused by an autoimmune disorder Higher risk of lymphoma Anti-double-stranded DNA (A) is highly specific for SLE, although it may also be seen in patients with RA. SLE is a chronic multisystemic autoimmune disease that most commonly affects women during their reproductive years and is characterized by the presence of positive antinuclear antibodies, oral ulcers, serositis, arthritis, photosensitivity, and malar and discoid rashes. Individuals with polymyositis often have positive anti-Jo-1 (B). Polymyositis is a rare autoimmune disease of the connective tissue characterized by the inflammation and weakness of skeletal muscles, particularly the muscles of the esophagus, lungs, and heart. Dermatomyositis is characterized by skin manifestations in addition to muscle inflammation and weakness. Anti-topoisomerase-1 (D) is often associated with scleroderma, an autoimmune disorder of connective tissues that causes progressive fibrosis of the skin and other organs. There are two types of scleroderma including localized scleroderma (morphea and linear scleroderma), which is more common in children and often manifests in the skin without any systemic involvement, and systemic scleroderma. Systemic scleroderma is further divided into limited systemic scleroderma, also known as CREST syndrome (calcinosis, Raynaud, esophageal dysmotility, sclerodactyly, and telangiectasias) and diffuse systemic cutaneous scleroderma. The limited form is associated with anti-centromere antibodies and the diffuse form is associated with anti-topoisomerase antibodies.

A 45-year-old man presents to the clinic with low back pain that radiates to his legs. He also has numbness and tingling in his legs. The symptoms began two months ago and have not progressed. Magnetic resonance imaging is obtained, which shows narrowing of the intraspinal canal. Which of the following is the best initial treatment? A. Epidural steroid injection and physical therapy B. Hydrocodone and physical therapy C. Ibuprofen and physical therapy D. Surgical intervention

C. Ibuprofen and physical therapy Spinal Stenosis Patient will be older Complaining of low back pain and stiffness when walking that is relieved when leaning forward Diagnosis is made by MRI Most commonly caused by narrowing of the lumbar spinal canal with compression of the nerve roots Management includes physical therapy, steroid injections, and/or surgery

A 38-year-old woman comes to the emergency department because of shortness of breath and cough, productive of blood-tinged sputum for 3 days. She has been unresponsive to antibiotic therapy from her family doctor. Her temperature is 38.0°C (100.4°F), pulse is 92/min, respirations are 18/min, and blood pressure is 110/68 mm Hg. Laboratory studies show an elevated erythrocyte sedimentation rate (ESR) and circulating cytoplasmic anti-neutrophil cytoplasmic antibodies (c-ANCA). Urinalysis shows dysmorphic red cells and red blood cell casts. Which of the following antibodies is most likely to be found in the patient's serum? A. Anti-centromere antibodies B. Anti-myeloperoxidase antibodies C. Anti-proteinase-3 antibodies D. Anti-dsDNA antibodies E. Anti-mitochondrial antibodies

C. Anti-proteinase-3 antibodies Granulomatosis with polyangiitis (GPA), formerly referred to as Wegener's granulomatosis (WG), is a systemic disorder that involves both granulomatosis and polyangiitis. It is associated with cytoplasmic anti-neutrophil cytoplasmic antibodies (c-ANCA), and anti-proteinase 3 antibodies. Classic signs and symptoms are hemoptysis, nasal mucosal ulceration, and saddle-nose deformity along with kidney damage (rapidly progressive glomerulonephritis, hematuria). Lung involvement that does not resolve with antibiotic therapy should raise a suspicious of a non-infectious etiology. Cytoplasmic antineutrophil cytoplasmic antibodies (c-ANCA) are associated with GPA and a specific subtype of these seen commonly in GPA are directed specifically against the anti-proteinase 3.

A 54-year-old woman comes to the clinic because of muscle weakness. She has noticed progressive difficulty with daily tasks, such as getting out of her car, walking up steps, getting up from a chair, and swallowing. Physical examination shows periorbital rash and edema, 3/5 strength in the proximal muscles of both the upper and lower limbs, and 5/5 strength in the distal extremities. Muscle biopsy is shown below. Which of the following best explains the mechanism of this patient's condition? A. Abnormal apoptosis and antibodies against proteins in the cell nucleus B. Abnormally excessive and rapid growth of the epidermal layer of the skin C. Autoantibodies binding to the vasculature D. Tyrosinase as an autoantigen affecting melanin E. Streptococcus pyogenes causing circumscribed and marginated rash

C. Autoantibodies binding to the vasculature Autoantibodies binding to the vasculature, muscle atrophy, and lymphocytic inflammation is the pathophysiology of dermatomyositis (DM), which is a systemic connective-tissue disorder related to polymyositis (PM). DM is characterized by skin rash, erythroderma, periungual telangiectasias, "mechanic's hands" (rough, cracked skin), centripetal flagellate erythema, symmetric proximal muscle weakness, pain, and dysphagia, among others.

A 40-year-old woman comes to the office because of worsening joint pain for 6 months. She has had joint pain for many years in both wrists, metacarpophalangeal (MCP) joints, and proximal interphalangeal (PIP) joints. Her distal interphalangeal (DIP) joints are spared. Her pain is always worse in the morning and improves as the day goes on. Lately she realizes both knees also hurt her and she is more tired than usual. Examination shows her painful joints to be mildly warm and her PIPs are flexed, while her DIPs are extended. Laboratory testing shows high titers of rheumatoid factor (RF), an elevated ESR and C-reactive protein, and a normocytic normochromatic anemia. Examination of the synovial fluid at the involved joints would most likely reveal which of the following? A. Clear fluid; 125 leukocytes/mm3; 20% polymorphonuclear leukocytes B. Clear-yellow fluid; 1,800 leukocytes/mm3; 20% polymorphonuclear leukocytes C. Clear-yellow fluid; 2,300 leukocytes/mm3; 60% polymorphonuclear leukocytes D. Reddish fluid; 1,500 leukocytes/mm3; 20% polymorphonuclear leukocytes E. Turbid, purulent fluid; 50,000 leukocytes/mm3; 80% polymorphonuclear leukocytes

C. Clear-yellow fluid; 2,300 leukocytes/mm3; 60% polymorphonuclear leukocytes Rheumatoid arthritis is a chronic inflammatory autoimmune disease that involves the synovium of the joints, causing damage to the cartilage and bone. Synovial analysis will show findings consistent with an inflammatory process (clear/yellow fluid; >2000 leukocytes/mm3; 50-70% polymorphonuclear leukocytes).

A 10-year-old boy is brought to his pediatrician's office by his parents because of worsening pain over his left lower leg for the past 2 months. He denies any recent trauma to his leg within the past year and has been generally healthy. Physical examination shows a discrete, hard, and exquisitely tender mass over the anterior aspect of the right tibia about 3 inches below the knee. ESR today is 20mm/hr. An X-ray of this patient's left leg is shown. Which of the following is the most likely diagnosis? A. Osgood-Schlatter disease B. Stress fracture C. Ewing's sarcoma D. Osteosarcoma E. Osteomyelitis

C. Ewing's sarcoma Ewing's sarcoma is a lytic lesion primarily affecting individuals in the second decade of life. This malignancy primarily occurs within the diaphyses of long bones. The most commonly affected bones are the femur, tibia, and humerus. The majority of Ewing sarcoma patients will also have a translocation of chromosome 11 and 22. Other translocations, such as t(21;22) and t(7;22) have also been reported with Ewing sarcoma. Patients with Ewing sarcoma may also show systemic symptoms such as a low grade fever, anemia, and an elevated erythrocyte sedimentation rate. Radiographs show a circular, lytic bone lesion with periosteal reaction (formation of new bone in the periosteum in reaction to a nearby insult). This pattern is often described as an "onion-skin" lesion.

A 65-year-old woman comes to the clinic because of gradual onset of pain in her right hand for several months that has become unbearable. She describes the pain surrounding the distal inter-phalangeal joint of her right second digit and her first carpo-metacarpal joint of the same hand. Her temperature is 37.1°C (98.8°F), pulse is 76/min, respirations are 16/min, and blood pressure is 136/72 mm Hg. Physical examination shows a limited range of motion at these joints, bony enlargement, and non-erythematous swelling present. Her pain and stiffness seem to worsen throughout the day. Which of the following is the most likely etiology of her pain? A. Acute trauma B. Disruption of bone remodeling C. Extracellular matrix breakdown D. Joint Inflammation E. Local infection

C. Extracellular matrix breakdown Osteoarthritis is characterized by pain and bony enlargement of often used joints and manifests as pain with joint usage and decreased joint range of motion. Morning stiffness may be present in osteoarthritis, but, in contrast to rheumatoid arthritis, the stiffness may worsen throughout the day, rather than improve.

A 64-year-old woman comes to the emergency department because of sinusitis, a runny nose, and hemoptysis for the past 3 months. The nasal drainage is often purulent and occasionally contains blood. The patient has also experienced new joint pains and fatigue during the same time-period. Examination shows tenderness to percussion over the maxillary sinuses; however, is otherwise non-contributory. Urinalysis shows 3+ microscopic haematuria and 2+ proteinuria. Chest X-ray is shown below. What is the most likely diagnosis? A. Chronic heart failure B. Goodpasture syndrome C. Granulomatosis with polyangiitis D. Non-Hodgkin lymphoma E. Pneumonia

C. Granulomatosis with polyangiitis Granulomatosis with polyangiitis (GPA) is an autoimmune condition that affects the lungs and kidneys. It is associated with cytoplasmic anti-neutrophil cytoplasmic antibodies (c-ANCA) and commonly causes nasal inflammation resulting in bloody or purulent nasal discharge.

Which of the following medications is most appropriate to treat the symptoms of arthritis in a patient with systemic lupus erythematosus? A. Cyclophosphamide B. Hydroxychloroquine C. Ibuprofen D. Prednisone

C. Ibuprofen Question: True or false: Antibodies to double-stranded DNA are absent in drug-induced lupus? Answer: True. Systemic Lupus Erythematosus (SLE) Patient often a woman Complaining of fever, lymphadenopathy, weight loss, general malaise, or arthritis PE will show malar rash ("butterfly rash") Labs will show anti-nuclear antibodies (ANA), anti-dsDNA antibodies, anti-Smith antibodies, anti-histone antibodies Treatment is NSAIDs, steroids, immunosuppressants, hydroxychloroquine Drug-induced lupus: Hydralazine, INH, Procainamide, Phenytoin, Sulfonamides (HIPPS) Best contraception - LNg-IUD or POP

A 12-year-old girl comes to the emergency department with her father because of right wrist pain. She states that the pain began 17 days ago and has continued to worsen. She reports being ill with a fever and an upper respiratory illness prior to the onset of the wrist pain. Her temperature is 38.5°C (101.3°F), pulse is 92/min, respirations are 22/min, and blood pressure is 132/76 mm Hg. Physical examination shows the right wrist is erythematous, warm, swollen, and tender to palpation over the distal radius. Radiographs of the wrist show soft tissue edema and a large radiolucency in the distal radius. Which of the following is the most definitive treatment for this condition? A. Biopsy of the bone lesion B. Excision of the bone lesion C. Intravenous antibiotics with irrigation and debridement lesion D. Intravenous antibiotics E. Irrigation and debridement of the bone lesion

C. Intravenous antibiotics with irrigation and debridement lesion Chronic osteomyelitis is defined as infection that has been present for more than 2 weeks and signs of devitalized bone. It is treated with surgical irrigation and debridement and intravenous antibiotics for 4-6 weeks.

A 16-year-old African-American boy comes to the clinic because of leg swelling for the past several months. His temperature is 37.1°C (98.8°F), pulse is 76/min, respirations are 14/min, and blood pressure is 118/76 mm Hg. Physical examination shows a large, tender soft tissue mass just proximal to the knee. He has lost 8 lbs since his last visit and has a hemoglobin level of 11.0 g/dL. Plain film demonstrates periosteal elevation with bone destruction. A biopsy is performed. Which of the following is most likely to be expected on the pathology specimen? A. Brown nodule surrounded by dense sclerotic cortical bone B. Dense normal bone with cartilaginous cap C. Large, necrotic, hemorrhagic mass with osteoid and bone D. Myxomatous tissue E. Neoplastic colon tissue

C. Large, necrotic, hemorrhagic mass with osteoid and bone Osteosarcoma is the most common primary bone malignancy in children. Osteosarcoma is more common in African-American males under the age of 20 years. The characteristic presentation of osteosarcoma is weeks to months of localized pain after an injury that waxes and wanes. Radiologic characteristics of a "sunburst pattern" are also commonly seen. The presence of a necrotic, hemorrhagic mass with osteoid and bone is the defining characteristic for all types of osteosarcoma. The most common sites of osteosarcoma in children are the shafts of long bones, especially the distal femur, the proximal tibia, and proximal humerus.

A 58-year-old man presents to clinic today for a routine follow-up after starting hydrochlorothiazide 25 mg by mouth daily for primary hypertension. The patient states that he had to go to the emergency department last week for excruciating toe pain and was diagnosed with gout. Which of the following agents can be added on to this patient's regimen to reduce the risk of future gout exacerbations? A. Chlorthalidone B. Furosemide C. Losartan D. Niacin

C. Losartan This patient is experiencing drug-induced hyperuricemia after the addition of hydrochlorothiazide. Thiazide diuretics (e.g., indapamide, chlorothiazide) cause hyperuricemia by both increasing renal reabsorption of uric acid in the renal tubule along with causing a relative volume depletion that further enhances uric acid reabsorption. Hyperuricemia caused by hydrochlorothiazide predisposes the patient to develop gout. Use of the angiotensin II receptor blocker losartan has been shown to reduce uric acid levels in patients receiving hydrochlorothiazide and may help prevent future gout exacerbations. Losartan lowers uric acid levels by reducing angiotensin II-induced reabsorption of uric acid in the renal tubules along with having a direct uricosuric effect that is unique to losartan. Additional benefits of adding losartan to this patient's regimen include mitigation of potassium losses caused by diuretic therapy and delivering additional antihypertensive effects. Losartan should be avoided in patients who are pregnant due to teratogenicity.

A 34-year-old Caucasian woman is referred to the office for the evaluation of a heart murmur. Her medical history is relevant to recently diagnosed systemic lupus erythematosus and type I diabetes mellitus. Upon medical interrogation, the patient claims to feel easily fatigued whenever she walks a few steps and feels extremely tired all day. On physical examination the patient has a holosystolic murmur, that is heard best at the apex, as well as, scattered petechiae in her conjunctiva, oral mucosa, and legs. Her temperature is 37.1°C (98.8°F), pulse is 72/min, respirations are 17/min, and blood pressure is 120/80 mmHg. She has had multiple blood cultures drawn by her physician, and all of them are finalized and showed no growth. At this point, this patient most likely has which of the following conditions? A. Pulmonary edema B. Mitral valve stenosis C. Mitral valve regurgitation D. Aortic stenosis E. Rupture of chordae tendineae

C. Mitral valve regurgitation The patient in the vignette is experiencing Libman-Sacks endocarditis, a non-bacterial thrombotic endocarditis that is characterized by sterile vegetations located in the mitral valve surface and chordae. This condition affects approximately 30 to 50% of individuals with systemic lupus erythematosus.

A 40-year-old woman comes to the office because she's been having worsening joint pain for 6 months. She has had joint pain for many years in both wrists, metacarpophalangeal (MCP) joints, and proximal interphalangeal (PIP) joints. Her distal interphalangeal (DIP) joints are spared. Her pain is always worse in the morning and improves as the day goes on. Lately she realizes both knees also hurt her and she is more tired than usual. Examination shows her painful joints to be mildly warm and her PIPs are flexed, while her DIPs are extended. Which of the following would most likely be seen on X-ray of the hands? A. Subchondral sclerosis B. Osteophytes C. Periarticular erosions D. Subchondral cysts E. Digit clubbing

C. Periarticular erosions Rheumatoid arthritis is an inflammatory autoimmune disease that involves the synovium of the joints, causing damage to the cartilage and bone. The radiographic features of rheumatoid arthritis can be remembered by the mnemonic LESS: Loss of joint space, Erosions, Soft tissue swelling, and Soft bones (osteopenia) The patient described likely has rheumatoid arthritis (RA). This chronic inflammatory autoimmune disease involves the synovium of the joints, causing damage to the cartilage and bone. The radiographic features of rheumatoid arthritis can be remembered by the mnemonic LESS: Loss of joint space, Erosions, Soft tissue swelling, and Soft bones (osteopenia) Women are more commonly affected and HLA-DR4 is associated. High rheumatoid factor (RF) is associated with severe disease, although 3% of the healthy population has RF. RA exhibits symmetrical arthritis of proximal joints (wrists, metacarpophlangeal (MCP), proximal interphalangeal (PIP), elbow, knee) often worse in the morning. Distal interphalangeal (DIP) joints are not affected. The patient here also exhibits Boutonniere deformities, commonly seen with RA. Synovial analysis will show findings consistent with an inflammatory process (clear/yellow fluid; <2000 WBC; 50-70% PMNs).

A 40-year-old man presents to the clinic with about two months of generalized fatigue and occasional fevers. He also reports unexplained weight loss of about 15 pounds in that time. Last week he started noticing more joint pain in his shoulders, hips, and knees and noticed some skin discoloration on his legs. Upon physical exam, the patient is afebrile but has a lacy, purplish-red rash on his lower extremities bilaterally that is non-tender to palpation and some ulcerations of the left ankle around the medial malleoli. You order a tissue biopsy of the area surrounding the ulcer. Which of the following is the most likely diagnosis? A. Erythema nodosum B. Peripheral vascular disease C. Polyarteritis nodosa D. Polymyositis

C. Polyarteritis nodosa Polyarteritis Nodosa Patient will be a man 40 - 50-years-old Complaining of malaise, fever, sore throat, joint and muscle aches and pains PE will show tender lumps under the skin, especially on the thighs and lower legs Labs will show ↑ ESR, ANCA negative Diagnosis is confirmed by biopsy showing necrotizing arteritis or by arteriography Treatment is steroids Comments: "starburst" livedo (painful violaceous plaques that are surrounded by livedo reticularis) is pathognomonic Which organ is usually unaffected in polyarteritis nodosa? Answer: The lungs, which distinguishes it from Wegener granulomatosis.

A 55-year-old woman presents to the clinic with several months of fatigue, bilateral shoulder pain, and bilateral hip pain. She states that she is achy and stiff in those areas, especially when she gets up in the morning. Only after lying in bed for about an hour can she get up and walk around. She does not complain of any headaches or scalp tenderness. Laboratory tests reveal an erythrocyte sedimentation rate of 80 mm/hr and a normal creatine kinase. Which of the following is the most likely diagnosis? A. Fibromyalgia B. Giant cell arteritis C. Polymyalgia rheumatica D. Polymyositis

C. Polymyalgia rheumatica Polymyalgia Rheumatica Patient will be a woman older than 50-years-old Complaining of proximal muscle (shoulders, neck, hip girdle) stiffness and aching especially in the morning, that improves during the day but worsens after inactivity PE will show symmetrical decreased range of motion in the neck, hips, and especially in the shoulder girdle Labs will show erythrocyte sedimentation rate (ESR) ≥ 40 mm/hour Treatment is low-dose prednisone Comments: Strongly associated with giant cell (temporal) arteritis

A 70-year-old man presents to the clinic with bilateral joint pain and morning stiffness. His shoulders and hips have the most prominent symptoms. The symptoms began abruptly three days ago. He denies any history of trauma. Physical examination reveals a decrease in range of motion but normal strength. Laboratory findings include an elevated erythrocyte sedimentation rate. Which of the following is the most likely diagnosis? A. Fibromyalgia B. Osteoarthritis C. Polymyalgia rheumatica D. Rheumatoid arthritis

C. Polymyalgia rheumatica Polymyalgia Rheumatica Patient will be a woman older than 50-years-old Complaining of proximal muscle (shoulders, neck, hip girdle) stiffness and aching especially in the morning, that improves during the day but worsens after inactivity PE will show symmetrical decreased range of motion in the neck, hips, and especially in the shoulder girdle Labs will show erythrocyte sedimentation rate (ESR) ≥ 40 mm/hour Treatment is low-dose prednisone Comments: Strongly associated with giant cell (temporal) arteritis

A 65-year-old woman comes to the office because of a 1-month history of fever, malaise, and weight loss. During the past 3 months, she has experienced pain and stiffness in her hips and shoulders, especially when she wakes up. She reports occasional unilateral headaches and worsening vision. She denies any history of head trauma. Neurological examination shows no abnormalities. Administration of which of the following is the most appropriate next step in management? A. Acetaminophen B. Ibuprofen C. Prednisone D. Infliximab E. Allopurinol

C. Prednisone Giant cell arteritis (GCA), which causes unilateral headache and jaw claudication, is often associated with polymyalgia rheumatica, an inflammatory condition that presents with aching and morning stiffness of the proximal joints. Due to the possible complication of permanent blindness, patients with suspected GCA should be treated immediately with glucocorticoids.

A 38-year-old man comes to the office because of ankle pain and skin changes that have ongoing for the past few months. He reports difficulty walking. He also states that his left index finger and right wrist felt as if they were sprained. One morning, the pain in his joints was so intense that he could hardly get out of bed. His primary care physician ordered ANA, ACPA, and RF titers, all of which came back negative. Physical examination shows swelling and erythema of several distal interphalangeal joints on both hands, and also shows scattered silvery plaques on the extensor surfaces of the skin. Which of the following is the most likely diagnosis? A. Gout B. Polymyositis C. Psoriatic Arthritis D. Rheumatoid Arthritis (RA) E. Systemic Lupus Erythematous (SLE)

C. Psoriatic Arthritis Psoriatic arthritis is a type of inflammatory joint disorder in the group of seronegative spondyloarthropathies. It is characterized as seronegative, due to the absence of rheumatoid factor (RF). It is also a disorder that is more common in individuals with tissue-type HLA-B27. Psoriatic arthritis is seen in a group of patients with the chronic skin disorder, psoriasis. Psoriasis is characterized by scaly skin lesions, most commonly at extensor surfaces. It can also affect the nails, and cause pitting, oncholysis - or the separation of the nail from the nail bed. Psoriatic arthritis is characterized by the involvement of axial and peripheral joints, especially the distal interphalangeal (DIP) joints of the hands and feet. The prolonged inflammation can lead to the swelling of the fingers or toes, also known as dactylitis.

A 20-year-old man presents to campus health because he has had a painful, swollen right knee. His knee swelling began three weeks ago. He also complains of itchy, red eyes and a left-sided lower back ache which began two weeks ago. He denies any trauma. His medical history includes treatment for a chlamydia infection six weeks ago. Family history is significant for paternal-sided ulcerative colitis. Examination shows bilateral conjunctival injection. The patient also walks with a limp, and has a large effusion of his right knee. Which of the following is the most likely diagnosis? A. Ankylosing spondylitis B. Psoriatic arthritis C. Reactive arthritis D. Rheumatoid arthritis E. Systemic lupus erythematosus

C. Reactive arthritis Reactive arthritis is an autoimmune condition that can occur after a Chlamydia infection. It is associated with the HLA-B27 haplotype and they rhyme "can't see, can't pee, can't climb a tree," is useful to remember the classical symptoms.

Which of the following drugs is safe in pregnancy & requires an ophthalmologic exam at baseline and every 2-6 months? a. MTX b. sulfasalazine c. leflunomide d. hydroxychloroquine

d. hydroxychloroquine Hydroxychloroquine inhibits locomotion of neutrophils & chemotaxis of eosinophils. It impairs complement dependent antigen antibody reactions. Takes 2-6 mo to benefit Common SE: GI + NVD Serious: retinal toxicity

A 65-year-old woman comes to the office for her annual exam. The patient explains that over the years she has been leaning forward more and more as she walks. Physical examination reveals an exaggerated lumbar lordosis with a significant step-off of the spinous process in the lumbar spine. Plain radiograph demonstrates 50% anterior displacement of L4 on L5. Which of the following most accurately describes this condition? A. Spinal stenosis B. Spondylitis C. Spondylolisthesis D. Spondylolysis E. Spondylosis

C. Spondylolisthesis Spondylolisthesis is characterized as the anterior displacement of a vertebral bone. It is often associated with spondylolysis, a unilateral or bilateral fracture in the pars interarticularis. A bilateral fracture is more likely to result in the slippage of the vertebral bone anteriorly relative to adjacent vertebrae bones. Common signs of spondylolisthesis include a general stiffening of the back and hamstrings that can effect both posture and gait. The low back pain feels worse with coughing and sneezing, and attempting to stand up. Compensatory leaning forward may also be seen. Symptoms can be managed with physical therapy and NSAIDS. However, in degenerative spondylolisthesis with spinal stenosis, spine surgery is often considered among older adults.

A 19-year-old ballerina comes to the office because of lower back pain during physical activity. She reports first feeling the pain 3 months ago as a dull ache, but states that her symptoms suddenly worsened 2 weeks ago. Musculoskeletal examination reveals an abnormally prominent spinous process of the 5th lumbar vertebra, and sharp pain in the lower lumbar region that radiates to both thighs during lumbar extension. The patient rates the pain as a 7 on a 10-point scale. Neurological examination reveals paresthesia over the lumbosacral area that also radiates to both thighs. An X-ray of the lateral lumbar spine is ordered and the following radiograph is obtained:Which of the following conditions is the most appropriate diagnosis of the patient based on presentation and imaging results? A. Spondyloarthritis B. Spondylodiscitis C. Spondylolisthesis D. Spondylolysis E. Spondylosis

C. Spondylolisthesis Spondylolisthesis presents with lumbar pain that radiates into the buttocks or the posterior legs. Additional sharp pains can be felt acutely during lumbar extension. Additional neurological symptoms such as numbness, paresthesias, and weakness can be felt in the lower limbs. Hyperlordosis and a posterior pelvic tilt can also be seen on physical examination. An anteroposterior and lateral X-ray are obtained to visualize the lumbar spine in patients with suspected spondylolisthesis, and additional magnetic resonance imaging can be ordered if the X-rays are unremarkable. Spondylolisthesis can be managed conservatively or by referral to a spinal surgeon. If motor deficits or evidence of cauda equina syndrome are observed, the patient should be referred to surgery immediately.

Which of the following diagnostic findings is suggestive of pseudogout? A. High serum uric acid B. Negatively birefringent crystals C. Synovial fluid analysis showing crystals with a rhomboid shape D. Synovial fluid analysis showing needle-like crystals

C. Synovial fluid analysis showing crystals with a rhomboid shape Pseudogout Complaining of monoarticular arthritis Labs will show rhomboid-shaped crystals, weakly positive birefringence Most commonly caused by calcium pyrophosphate crystals Management includes intra-articular steroid injection (2 or less joints), NSAIDs, colchicine, and systemic corticosteroids Pseudogout, also called calcium pyrophosphate deposition disease, is a common crystal-induced arthritis that generally affects the large joints. It is most common in women over age 65. Most cases are idiopathic. The clinical findings are similar to those seen in acute gout. Pseudogout presents with recurrent inflammatory arthritis, which manifests as intense pain, swelling, erythema, and warmth. It is often monoarticular, but more than one joint can be involved. The knee is the most common joint affected in pseudogout, whereas the metatarsophalangeal joint of the great toe is the most common joint affected in gout. The diagnosis of pseudogout is largely based on synovial fluid analysis. Synovial fluid is collected by arthrocentesis, or joint aspiration, of an affected joint. The classic synovial fluid finding for pseudogout is weakly positively birefringent, rhomboid-shaped calcium pyrophosphate crystals. Radiographic evidence of pseudogout is also supportive. Imaging of the affected joint should be obtained to evaluate for cartilage calcification, which is also called chondrocalcinosis. The preferred treatment for pseudogout is an intraarticular glucocorticoid injection. However, oral medications, including nonsteroidal anti-inflammatory drugs (NSAIDs), glucocorticoids, and colchicine, are also used in acute management. For patients with frequent pseudogout attacks, colchicine is used prophylactically to prevent attacks.

Upon examination of your patient's odontoid view Xray, you see a fracture of the dens that extends into the cancellous body of C2. What type of fracture is this? A. Type I B. Type II C. Type III D. Type IV

C. Type III Explanation: Type I: Fx of tip of odontoid Type II: Fx through the base (unstable) Type III: Fx extends into cancellous body of C2 (unstable) *Types II & III do NOT heal well, interruption of blood supply - need intervention

Which of the following physical examination findings is most likely in a patient with a herniated nucleus pulposus at the L4-5 level? A. Absent ankle jerk reflex B. Absent knee jerk reflex C. Weakness of ankle dorsiflexion D. Weakness of ankle plantar flexion

C. Weakness of ankle dorsiflexion An absent ankle jerk reflex (A) is seen in a patient with an S1 radiculopathy. An absent knee jerk reflex (B) is characteristic of an L4 radiculopathy and is also seen with L2 and L3 radiculopathies. Weakness of ankle plantar flexion (D) occurs in S1 radiculopathy, as this nerve root supplies the gastrocnemius and soleus muscles. Herniated Disk Pulposus Patient presents with abrupt onset of pain that is severe and exaggerated by sitting, walking, standing, or coughing Pain radiates from the buttock down the posterior or posterolateral leg to the ankle or foot PE will show positive straight leg raise, tenderness in the sciatic notch, and limited ROM Diagnosis is made by MRI Most commonly caused by herniation of the nucleus pulposus into the lumbar spinal canal Most common location L4 to L5 or L5 to S1

Which of the following drugs may cause more cutaneous SE's, particularly in Asian populations, due to having HLA-B5801? a. pegloticase b. colchicine c. indomethacin d. prednisone e. allopurinol

e. allopurinol

Scleroderma is caused by _____________ activation leading to collagen deposition, resulting from autoimmune damage to mesenchymal tissue.

fibroblast

A 65-year-old woman comes to the office because of a 3-month history of neck and shoulder pain. She initially noticed symptoms only on the left side but has experienced bilateral pain and stiffness for the past two months. The stiffness is worst in the morning and lasts more than an hour. She reports a 4.5-kg (10-lb) weight loss during the past three months. Physical examination shows swelling of shoulders with limited range of motion. Muscle strength is normal in all extremities. Laboratory studies show an erythrocyte sedimentation rate of 70 mm/h. Which of the following is the most appropriate next step in management? A. X-rays of the shoulders B. Measurement of creatine kinase levels C. Temporal artery biopsy D. Administration of prednisone E. Muscle biopsy

D. Administration of prednisone This patient's symptoms of aching and morning stiffness in the neck and shoulders are consistent with polymyalgia rheumatica (PMR), a chronic inflammatory disease of unknown cause that typically affects individuals older than 50 years of age. PMR is characterized by pain and morning stiffness in the neck, shoulders, torso, and hips. Other symptoms include distal musculoskeletal manifestations like synovitis, tenosynovitis, and bursitis. Nonspecific features include fever, malaise, weight loss, and normocytic anemia. Laboratory studies show an elevated erythrocyte sedimentation rate. PMR can be treated with low-dose glucocorticoids. High-dose treatment is appropriate for patients with concurrent giant cell arteritis, to prevent the potential complication of vision loss. PMR is diagnosed clinically, but temporal artery biopsy should be performed in patients with suspected giant cell arteritis.

A 55 year-old man comes to the emergency department because of a three-day history of pain, erythema, and swelling in the first metatarsophalangeal joint of his right foot. He is unable to sleep at night because he has severe pain whenever his bed sheet touches the joint. He admits to smoking one pack per day and drinking three to four beers nightly. He denies any previous episodes of pain in this joint and denies trauma to the area. He denies pain in any other joint. His only medication is hydrochlorothiazide for hypertension, and the dose of this medication was recently increased by his primary care provider. Physical examination shows a tender, warm, erythematous right toe. Synovial fluid is removed from the joint, and crystals are seen on microscopy. Which of the following substances is most likely to be seen on analysis? A. Calcium oxalate B. Calcium pyrophosphate dihydrate C. Cholesterol D. Monosodium urate E. Uric acid

D. Monosodium urate Patients with gout are characterized by high levels of uric acid, but it is the monosodium urate crystals deposition within the joints that causes gout and is seen on joint aspiration analysis.

A 52-year-old female presents acutely with podagra and is prescribed indomethacin. Several weeks later, her physician plans to prescribe her a xanthine oxidase inhibitor to help prevent future exacerbations. Which of the following would most likely increase the risk of this patient developing an adverse cutaneous reaction to this new drug? A. Presence of the HLA-B*1502 allele B. Co-administration of a loop diuretic C. Liver disease D. Allopurinol dosage of 300 mg/day

D. Allopurinol dosage of 300 mg/day It is important to remember that allopurinol can cause Steven-Johnson syndrome/Toxic epidermal necrolysis. A dose of 300 mg/day is way too high of a dose to initially put a patient on. You can titrate the dose up every 2-4 weeks. The HLA-B*5801 allele has been found in many patients who have been diagnosed with drug hypersensitivity, SJS or TEN from allopurinol therapy. At this time, patients who should be screened for the presence of this haplotype before starting allopurinol are those of Chinese or Thai ancestry, or of Korean descent with stage 3 chronic kidney disease or greater. In these populations, HLA-B*5801 is ubiquitous and has been closely linked to adverse drug reactions. According to a 2011 study, patients of Japanese and European ancestry with the HLA-B*5801 allele may also be at risk, but screening for those patients is not recommended at this time (August 2014). Another urate-lowering drug, feboxustat, may be used instead. Major Takeaway: Higher doses of allopurinol, renal insufficiency and co-administration of thiazide diuretics all increase the risk of adverse reactions to allopurinol. Allopurinol should be discontinued at the first sign of a rash or hypersensitivity syndrome.

A 38-year-old woman comes to the emergency department because of shortness of breath and a cough productive of blood-tinged sputum for 3 days. She had a similar episode several weeks ago and was prescribed antibiotics by her family doctor with minimal improvement. Over the past 2 months, she has experienced mild fever, malaise, and weight loss as well as persistent red-tinged rhinorrhea. She does not smoke cigarettes or use illicit drugs. Temperature is 38.0°C (100.4°F), pulse is 92/min, respirations are 18/min, and blood pressure is 110/68 mm Hg. Physical examination shows nasal crusting and multiple ulcer formations in the nasal cavity. Laboratory studies show an elevated erythrocyte sedimentation rate (ESR). Urinalysis shows dysmorphic red cells and red blood cell casts. Which of the following antibodies is most likely to be found in the patient's serum? A. Anti-centromere antibodies B. Anti-myeloperoxidase antibodies C. Anti-dsDNA antibodies D. Anti-proteinase-3 antibodies E. Anti-mitochondrial antibodies

D. Anti-proteinase-3 antibodies This patient with a history of fatigue, weight loss, and persistent bloody rhinorrhea is now presenting with shortness of breath and a productive bloody cough. Her presentation, in conjunction with the urinalysis results and unresponsiveness to antibiotic therapy, is consistent with granulomatosis with polyangiitis (GPA). GPA, formerly referred to as Wegener granulomatosis, is a systemic autoimmune disorder that belongs to the antineutrophil cytoplasmic autoantibody (ANCA)-associated vasculitides (together with microscopic polyangiitis and eosinophilic granulomatosis with polyangiitis). It mainly affects small- and medium-sized vessels throughout the body. Classic signs and symptoms are involvement of 3 systems: the upper respiratory tract (e.g. saddle nose deformity, nasal mucosal ulceration, otitis media), lower respiratory tract (e.g.. hemoptysis, lung nodules), and the kidneys (glomerulonephritis, hematuria). Lung involvement that does not resolve with antibiotic therapy should raise a suspicion of a non-infectious etiology. Patients with vasculitis have circulating antibodies that react with neutrophil cytoplasmic antigens termed "antineutrophil cytoplasmic antibodies" (ANCAs). ANCAs are a heterogeneous group of autoantibodies directed against neutrophil primary granules elements (mainly enzymes). Anti-proteinase-3 (PR3-ANCA), previously called c-ANCA, is a neutrophil azurophilic granule component that shares similar structure with numerous microbial peptides, possibly explaining the generation of these antibodies. PR3-ANCAs are associated with granulomatosis with polyangiitis.

A 20-year-old man presents to the office because of joint pain, particularly in his fingers. Physical examination shows significant swelling of his fingers and pitting in his fingernails. He is most tender to palpation across his DIP joints. Three months ago, he visited the office because of a rash, shown below. Besides some itching, he denied any other symptoms. Topical treatments helped with the itching. Which of the following haplotypes is most commonly associated with this condition? A. HLA-DQ8 B. HLA-DR5 C. HLA-DR3 D. HLA-B27 E. HLA-DR4

D. HLA-B27 Psoriatic arthritis is a seronegative spondylarthopathy, like reactive arthritis and ankylosing spondylitis, and is associated with the HLA-B27 haplotype.

A 4-year-old boy is brought to the emergency department by his mother because of a 1 day history of rash. She says that he had Streptococcal pharyngitis several weeks ago, but was in his normal state of health until 5 days ago when he began complaining of right hip pain. This subsided and was replaced by right knee and ankle pain one day ago. At the same time, he developed a rash of dark red spots scattered on his legs, buttocks, and forearms. He also began complaining of nausea and stomach pains 2 hours ago. Physical examination shows a purpuric rash located in dependent areas, and a swollen right knee without effusion, erythema, or warmth. Urinalysis shows microscopic hematuria with proteinuria. Serum electrolytes, including creatinine, are within normal limits. Which of the following is the most likely diagnosis? A. Reactive arthritis B. Rheumatic fever C. IgA nephropathy D. IgA Vasculitis E. Systemic lupus erythematosus

D. IgA Vasculitis

Which of the following pharmacologic therapies is generally a first-line treatment for osteoarthritis? A. Acetaminophen B. Hyaluronic acid C. Hydrocodone D. Naproxen

D. Naproxen Osteoarthritis Patient will be an older individual Hip or knee pain X-ray will show subchondral sclerosis, joint space narrowing, subchondral cysts, and osteophytes Management includes NSAIDs, acetaminophen (lack of benefit), joint injections, physical therapy, joint replacement No constitutional symptoms Bony enlargement is seen at the distal interphalangeal (DIP) joints and proximal interphalangeal (PIP) joints called Heberden nodes and Bouchard nodes respectively. Osteoarthritis can be diagnosed without the use of radiography in the presence of typical symptoms and signs in patients who are within the at-risk age group. Laboratory testing and imaging can be used in atypical cases. Laboratory testing may include inflammatory markers, such as erythrocyte sedimentation rate (ESR) and C-reactive protein (CRP). These are normal in osteoarthritis and help rule out other conditions. Radiographic findings can support osteoarthritis and may reveal joint space narrowing, osteophytes formation, thickened, dense subchondral bone, and bone cysts

A 24-year-old man presents to his primary care physician complaining of low back pain for the last four months. The pain is worse in the mornings and after periods of inactivity. He is otherwise healthy with no recent trauma or illnesses. His father was diagnosed with ankylosing spondylitis around the same age. Which of the following is the best first test to confirm this diagnosis? A. C-reactive protein B. HLA-B27 C. Lumbar X-ray D. Pelvic X-ray

D. Pelvic X-ray Ankylosing Spondylitis Patient will be a man Low back pain that's most severe at night and morning stiffness that improves with exercise or movement X-ray will show multiple vertebral fusions (bamboo spine) Labs will show increased ESR Most commonly caused by the presence of HLA-B27 Treatment options include NSAIDs, physical therapy, TNF-alpha blockers Diseases associated with HLA-B27: PAIR Psoriatic arthritis Ankylosing spondylitis Inflammatory bowel disease Reactive arthritis

A 55-year-old woman presents with increasing weakness in her muscles that began in her legs and then progressed to her shoulders and neck for the past six months. Her physical exam reveals proximal muscle weakness of her hips, shoulders, and neck, which are non-tender to palpation. A blood test reveals an elevated creatine kinase and an elevated antinuclear antibody. A muscle biopsy is ordered to confirm diagnosis. Which of the following is the most likely diagnosis? A. Cushing syndrome B. Hypothyroidism C. Polymyalgia rheumatica D. Polymyositis

D. Polymyositis Polymyositis is a systemic disease that is primarily characterized by muscle weakness. It affects all age groups but peaks in the fifth and sixth decade of life. It is more common in women than men and more common in blacks than in whites. Patients will present classically with progressive bilateral proximal muscle weakness over the course of weeks to months. The muscles involved typically include the muscles in the neck and upper and lower extremities. Lower extremity symptoms usually occur before upper extremity symptoms. Some patients may have muscle pain or tenderness. Dysphagia and interstitial lung disease are also common symptoms. As the disease progresses, atrophy of the muscles can occur, as well as contractures of the joints. Laboratory tests will reveal elevated creatine kinase as well as elevated lactate dehydrogenase (LD), aldolase, aspartate aminotransferase (AST) and alanine aminotransferase (ALT). Testing may also reveal an elevated antinuclear antibody level. Specific antibody tests are positive in certain types of polymyositis and include anti-Jo-1 antibodies, anti-SRP antibodies, and anti-Mi-2 antibodies. Anti-Jo-1-antibodies are the most common myositis-specific autoantibodies found and are highly correlated with the development of interstitial lung disease. Muscle biopsy is the only specific diagnostic test and will demonstrate myopathic inflammatory changes. Corticosteroids are the mainstay of treatment.

A 62-year-old woman presents to the office with scalp tenderness on the left side of her head, double vision, and pain with chewing that started about two days ago and has gotten worse. Physical exam reveals a tender and enlarged left temporal artery, and you suspect giant cell arteritis. Which of the following is the most appropriate treatment for this condition? A. Aspirin B. Methotrexate C. Nonsteroidal anti-inflammatories D. Prednisone

D. Prednisone Temporal Arteritis (Giant Cell Arteritis) Patient will be a woman > 50 years old Monocular visual loss, unilateral headache, jaw claudication PE will show a tender temporal artery Labs will show ESR > 50 mm/hour Diagnosis is made by temporal artery biopsy Treatment is high-dose steroids ASAP Associated with polymyalgia rheumatica

A 62-year-old woman comes to the office because of pain in her hands for three years. She describes the pain as progressive - coming on slowly and worsening over the course of the three years. She says that her aunt and wheelchair-bound grandmother had problems with their hands as well. A hand X-ray is obtained which shows soft tissue swelling and marked juxta-articular osteopenia in her metacarpophalangeal and proximal interphalangeal joints, and minor bony erosions. Which of the following will most likely be present on her plain film ten years from now? A. No further progression B. Pencil-in-cup deformity, ankylosis, periostitis, dactylitis C. Subchondral sclerosis, subchondral cysts, osteophytes, joint space narrowing D. Subluxations (e.g. boutonniere and swan-neck deformities), joint ankylosis, z-thumb deformity E. Subperiosteal bone resorption

D. Subluxations (e.g. boutonniere and swan-neck deformities), joint ankylosis, z-thumb deformity Rheumatoid arthritis characteristic early signs include soft tissue swelling, osteopenia, erosions, and joint-space narrowing. Later changes include subluxation causing ulnar deviation, z-thumb, boutonniere and swan neck deformities, and ankylosis.

Which of the following is the anatomical finding seen in patients with spondylolisthesis? A. Degeneration of the vertebra B. Sideways curvature of the spine C. Vertebra slipping backward with respect to the vertebra below D. Vertebra slipping forward with respect to the vertebra below

D. Vertebra slipping forward with respect to the vertebra below Spondylolisthesis is when a vertebra slips forward with respect to the vertebra below. It can occur anywhere on the spine but is most commonly seen in the lower lumbar region. Spondylolisthesis is two times more common in men than women. Other risk factors include the presence of scoliosis, a first-degree relative with the condition, and occult spina bifida at S1. There are six types of spondylolisthesis: congenital, isthmic, degenerative, traumatic, pathologic, and postsurgical. The majority of cases are believed to be due to overuse, specifically due to hyperextension of the lumbar spine. Most patients with spondylolisthesis are asymptomatic. When symptoms occur, low back pain is the most common complaint. Red flag symptoms to watch for include a history of trauma, incontinence, fever, unintentional weight loss, intravenous drug use, intense localized pain and difficulty with getting into a comfortable position.

A 57-year-old Caucasian man presents to his primary care physician for two days of fever and cough with shortness of breath. A chest X-ray demonstrates a right lower lobe pneumonia. There is also an incidental finding of lytic lesions in the first lumbar vertebral body consistent with Paget disease. Which of the following is the best test for initial diagnosis of this condition? A. Bone biopsy B. CT Scan C. Radionuclide bone scan D. X-ray

D. X-ray Paget Disease of Bone (PDB, Osteitis Deformans) Labs will show increased serum alkaline phosphatase and bone-specific alkaline phosphatase X-ray will show bone thickening and enlargement with thickened cortices Most commonly caused by an increase in osteoclastic activity followed by an increase in osteoblastic activity Treatment is bisphosphonates

A heliotrope rash and Gottron's papules are seen frequently in (dermatomyositis/polymositis) _____________.

Dermatomyositis

(Diffuse/Limited) ___________ scleroderma is associated with widespread skin involvement, rapid progression and early visceral involvement.

Diffuse

(Diffuse/Limited) ____________ scleroderma is associated with anti-Scl-70 (anti-DNA topoisomerase I) autoantibodies.

Diffuse

A 62-year-old woman comes to the clinic for a review appointment because of lower limb pain. She reports that she has no pain elsewhere in her body. She has no contributory medical history and is not on any regular medications. An X-ray was ordered at her last appointment. Based on the radiographic image shown, which of the following signs and symptoms are most likely to be present? A. Lateral hip pain and tenderness over the greater trochanter B. Pain radiating below the knee associated with relief during lumbar flexion C. Night-time hip pain with unintended weight loss and general fatigue D. Severe pain and an inability to ambulate due to traumatic injury E. Groin pain associated with significant loss of internal rotaton of the hip

E. Groin pain associated with significant loss of internal rotaton of the hip Osteoarthritis of the hip typically causes groin pain and impaired internal rotation of the affected hip. Characteristic radiographic findings are joint space narrowing and osteophyte formation.

A 26-year-old woman comes to the office because of a 1-week history of foreign body sensation in both eyes and eye redness. Her medical history is relevant to a recent and uncomplicated laser refractive eye surgery. The patient complains that for the past few days, she wakes up in the morning with a whitish mucoid discharge in both eyes and a blurry vision. Eye exam shows corneal xerosis and conjunctival redness. Her temperature is 36.7°C (98.2°F), pulse is 61/min, respirations are 16/min, blood pressure is 110/60 mmHg. Which of the following is the most likely diagnosis? A. Vernal keratoconjunctivitis B. Bacterial conjunctivitis C. Viral conjunctivitis D. Allergic conjunctivitis E. Keratoconjunctivitis sicca

E. Keratoconjunctivitis sicca (dx: Sjogren's) The patient in the vignette is experiencing keratoconjunctivitis sicca, a dry eye syndrome characterized by symptoms and signs such as irritation, redness, eye itching, and blurred vision. Keratoconjunctivitis sicca is an inflammatory and degenerative disorder associated with a deficient production of the ocular tear film. Patients often complain of a sandy feeling under their eye, because of decreased lacrimal production.

A 3-year-old boy is brought to the emergency department by his parent due to a fever for the past six days. The fever has not responded to treatment with acetaminophen. His parent mentions that the patient has refused to eat due to pain in his mouth. During his bath yesterday, the parent also noticed that he has a rash covering his trunk and limbs. Temperature is 39°C (102.2°F), pulse is 125/min, and blood pressure is 90/55 mm Hg. Examination shows an irritable-appearing child with a polymorphous rash covering the trunk which peels off near the fingers, as well as bilateral eye injection without exudate. Cervical lymphadenopathy is present. Which of the following complications may be seen in this patient if left untreated? A. Subacute sclerosing panencephalitis B. Reye syndrome C. Staphylococcal scalded skin syndrome D. Glomerulonephritis E. Mitral regurgitation F. Hemorrhagic cystitis

E. Mitral regurgitation The main complication seen in Kawasaki disease is coronary artery abnormalities, which may include dilation, aneurysm, and/or stenosis. During the acute phase, patients may develop depressed ventricular function, valvular regurgitation, and pericardial effusion. Late complications consist of myocardial infarction, arrhythmias, stenosis, and occlusion. Acute myocardial infarction is the main cause of death in Kawasaki disease, and early treatment with intravenous immunoglobulin (IVIG) and aspirin is important to avoid this complication. Kawasaki disease is one of the few cases where it is acceptable to treat children with aspirin (generally aspirin is avoided in children due to the risk of developing Reye syndrome).

A 20-year-old man comes to the clinic due to the recent appearance of skin lesions. The patient's history is significant for chronic asthma, recurrent sinusitis, and nasal polyposis. On physical examination, tender subcutaneous nodules are present on the bilateral shins. The results of the white blood cell differential are shown below: Laboratory value Result Leukocyte count 16,000/mm3 Neutrophils, segmented 44% Neutrophils, banded 5% Eosinophils 29% Basophils 0% Lymphocytes 15% Monocytes 7% This patient's condition is associated with antibodies directed against which of the following? A. Mitochondria B. Basement membrane C. Transglutaminase enzyme D. Microsome E. Myeloperoxidase enzyme (MPO-ANCA) F. Proteinase

E. Myeloperoxidase enzyme (MPO-ANCA) Eosinophilic granulomatosis with polyangiitis (Churg-Strauss) is an autoimmune small-vessel vasculitis characterized by asthma and marked eosinophilia. Patients are often found positive for anti-myeloperoxidase (MPO-ANCA), a lysosomal granule constituent which is involved in oxygen free radical generation.

A 10-year-old girl comes to the emergency department because of left ankle pain. She states that the pain started several weeks ago and became progressively worse. The patient also reports fevers and diffuse joint pain. Her temperature is 39.5°C (103.1°F), pulse is 102/min, respirations are 24/min, and blood pressure is 136/72 mm Hg. Physical examination reveals the left ankle is erythematous, warm, swollen, and tender to palpation over the medial malleolus. Radiographs of the ankle show soft tissue edema and a large radiolucency in the distal tibia. Which of the following is the most appropriate next step in management? A. Biopsy of the bone lesion B. Bone scan of the lower extremity C. Excision of the bone lesion D. Irrigation and debridement of the bone lesion E. Obtain a blood sample for CRP, ESR, and culture

E. Obtain a blood sample for CRP, ESR, and culture Blood should be collected to analyze the C-reactive protein (CRP), erythrocyte sedimentation rate (ESR), and blood cultures to evaluate inflammation and systemic infection. Ultimately, the treatment consists of IV antibiotics, irrigation, and debridement of the bony lesion. The timing of antibiotics relative to debridement and culture typically varies from patient to patient.

A 14-year-old girl comes to the clinic because of a referral by a school nurse for a curve in her spine. The patient denies shortness of breath, cardiac palpitations, or other significant medical history. Her temperature is 37.1°C (98.8°F), pulse is 70/min, respirations are 16/min, and blood pressure is 114/72 mm Hg. Physical examination shows a spinal curve with an estimated Cobb angle to be approximately 12°. The spinal curve ranges laterally from T7-T12, with the convexity of the curve to the left, resulting in a levoscoliosis. Which of the following is the most appropriate imaging study to perform in this patient? A. CT of the spine B. Lateral radiograph of the spine during flexion C. MRI of the spine D. Oblique radiograph of the spine E. Standing AP and lateral radiographs of the spine

E. Standing AP and lateral radiographs of the spine The most appropriate imaging study to order inpatients suspected of having scoliosis is a standing AP and lateral X-ray of the spine.

A 15-year-old man comes to his pediatrician's office because of pain in his right humerus for the past 3 months. He also reports a recent low grade fever. Further examination reveals midshaft swelling on his right humerus. The patient denies any recent history of trauma. Genetic analysis shows an 11;22 translocation. An x-ray of his right arm is taken. Which of the following answer choices will most likely show up on imaging for this patient? A. Spiral fracture B. Necrosis surrounded by sclerosis of bone C. Punched-out lytic lesions D. Codman triangle E. Onion-skin appearance

E. Onion-skin appearance Primary bone malignancies are disease which are primarily found in children. The most common primary bone malignancies in the pediatric population is osteosarcoma and Ewing sarcoma. In this clinical vignette, the most likely diagnosis is Ewing sarcoma. Patients with Ewing sarcoma are classically males younger than 15 years of age with a low grade fever. The most common translocation in Ewing sarcoma is t(11;22). However, t(21;22) and t(7;22) are other translocations which have been reported with Ewing sarcoma. Histologically, Ewing sarcoma has the appeaPrimary bone malignancies are disease which are primarily found in children. The most common primary bone malignancies in the pediatric population is osteosarcoma and Ewing sarcoma. In this clinical vignette, the most likely diagnosis is Ewing sarcoma. Patients with Ewing sarcoma are classically males younger than 15 years of age with a low grade fever. The most common translocation in Ewing sarcoma is t(11;22). However, t(21;22) and t(7;22) are other translocations which have been reported with Ewing sarcoma. Histologically, Ewing sarcoma has the appearance of small, round blue cells that are look lymphocytic in appearance. These tumor cells are derived from the neuroectoderm, and often metastasizes.

A 65 year-old woman comes to the emergency department because of extreme pain in her right knee. She states she was grocery shopping when the pain began, but she does not know what caused it. She has a history of Paget's disease for five years. Physical examination shows a limited range of motion of her right knee and imaging reveals a fracture of the tibia. A bone biopsy specimen shows a loss of normal trabeculae, with a mosaic pattern and increased numbers of osteoclasts and osteoblasts. Which of the following is the most likely diagnosis? A. Ankylosing spondylitis B. Enchondroma C. Fibrous dysplasia D. Osteoid osteoma E. Osteosarcoma

E. Osteosarcoma Osteosarcomas can arise in approximately 5-10% of patients with severe Paget disease of the bone and all patients with Paget are at increased risk for the neoplasm. The tumor is most common in the long bones (distal femur, proximal tibia, proximal humerus). The symptoms of osteosarcoma are characterized by bone fracture, pain, limitation of motion, limping, tenderness, swelling, and redness at the site of the tumor. Osteosarcomas can thus lead to pathological fractures in the affected bones. Osteosarcomas are thought to arise from actively remodeling bone and thus appear as normal trabeculae with a mosaic pattern and production of tumor osteoid and bone on microscopic analysis. Treatment for non-metastatic osteosarcoma is based on surgery and systemic chemotherapy

A 33-year-old man comes to the clinic because of new skin lesions for the past 9 months. The patient says that the skin lesions have been growing in number and that they tend to blister and then form a black center. Medical history is contributory for asthma for which he requires an albuterol inhaler as needed. Physical examination shows bleeding areas on the arms and legs with a central black dimple. Laboratory test shows 11% of eosinophils in the blood. Which of the following serologies is most likely to be positive or elevated in this patient's condition? A. Anti-Ro B. Anti-Sm C. C-ANCA D. HBsAg E. P-ANCA

E. P-ANCA Eosinophilic granulomatosis with polyangiitis (EGPA; also known as Churg-Strauss syndrome [CSS] or allergic granulomatosis) is a rare autoimmune condition that causes vasculitis in persons with a history of airway allergic hypersensitivity (atopy). CSS is characterized by three stages: the early (prodromal) stage is marked by airway inflammation; almost all patients experience asthma and/or allergic rhinitis. The second stage is characterized by hypereosinophilia, which causes tissue damage, most commonly to the lungs and the digestive tract. The third stage consists of vasculitis, which can eventually lead to cell death and can be life-threatening. Laboratory analysis shows eosinophilia and raised immunoglobulin E levels. 50-60 % of patients are ANCA positive, predominantly p-ANCA. Rheumatoid factor is also positive in 70 % of patients. The American College of Rheumatology 1990 criteria for diagnosis of Churg-Strauss syndrome lists these criteria: asthma, eosinophils greater than 10% of a differential white blood cell count, mononeuropathy or polyneuropathy, unfixed pulmonary infiltrates, paranasal sinus abnormalities, and histological evidence of extravascular eosinophils. Treatment for CSS includes glucocorticoids (e.g. prednisolone) and other immunosuppressive drugs (e.g. azathioprine and cyclophosphamide). In many cases, the disease can be put into a type of chemical remission through drug therapy, but the disease is chronic and lifelong.

A 52-year-old man comes to the emergency department because of joint swelling, pain, and trouble walking. Physical examination shows his right knee is swollen and tender to palpation. An aspiration of the joint shows calcium pyrophosphate crystal accumulation. Which of the following findings will most likely be found upon microscopic examination of the synovial fluid? A. Elevated eosinophils B. Envelope-shaped crystals C. Less than 1,000/uL white blood cells D. Negatively birefringent needle shaped crystals E. Positively birefringent rhomboid crystals

E. Positively birefringent rhomboid crystals The joint aspirate of a patient with chondrocalcinosis, also known as psuedogout, will be characterized by an elevated WBC count and the positively birefringent rhomboid shaped crystals of calcium pyrophosphate.

A 55-year-old woman comes to the clinic because of increasing difficulty climbing stairs and getting out of chairs for the past 6 months. She denies fatigue, muscle aches, weight loss, fever, or joint pain. She does not take any medications and denies the use of drugs or alcohol. Physical examination shows a reddish-purple discoloration and swelling of the eyelid, and a scaly, erythematous rash over the knuckles. Which of the following is the most appropriate next step in management? A. ANA test B. Electromyography C. Erythrocyte sedimentation rate D. Muscle biopsy E. Serum creatine kinase

E. Serum creatine kinase Dermatomyositis (DM) is a systemic connective-tissue disorder related to polymyositis (PM) characterized by skin rash (Gottron's sign, eyelid "lilac" rash, and shawl or V-sign), erythroderma, periungual telangiectasias, "mechanic's hands" (rough, cracked skin), scalp soriaform changes, centripetal flagellate erythema, calcinosis cutis, symmetric proximal muscle weakness, pain, tenderness, temporary paralysis, and dysphagia, among others. ESR and CK are elevated and there are EMG findings of spontaneous muscle fibrillation and short polyphasic muscle potentials. The most appropriate first step for diagnosis is the study of symptomatology and CK blood measurements, because it is a muscle enzyme that is released into the serum when muscles are damaged. Definitive diagnosis is through muscle biopsy and skin biopsy and EMG are complementary. The cause is unknown, but it may result from an initial viral or bacterial infection and the mechanism is conjectured to be complement-mediated damage of microscopic vessels with muscle atrophy and lymphocytic inflammation secondary to tissue ischemia. DM is treated with steroids, chemotherapy, specialized exercise therapy, and medications (e.g. prednisolone, methotrexate, mycophenolate, and hydroxychloroquine, among others).

A 15-year-old girl comes to the emergency department because of left knee pain. She states the pain has been increasing for the past 2 weeks. She also reports fevers, diffuse joint pain, and a history of sickle cell disease. Her temperature is 38.3°C (100.9°F), pulse is 88/min, respirations are 20/min, and blood pressure is 122/82 mm Hg. Physical examination shows the left knee is erythematous, warm, swollen, and tender to palpation over the tibial prominence. Radiographs of the knee show soft tissue edema and a large radiolucency in the proximal tibia. Which of the following is the most likely infectious organism? A. Group B streptococcus B. Haemophilus influenzae C. Legionella pneumophila D. Pseudomonas aeruginosa E. Staphylococcus aureus

E. Staphylococcus aureus Osteomyelitis refers to bony inflammation and is almost always due to infection, typically bacterial. It is particularly common between the ages of 2 and 12 and three times more common in males than females. Even among pediatric patients with sickle cell disease, Staphylococcus aureus is the most common cause of osteomyelitis in all demographic groups. After S. aureus Escherichia coli is relatively common in intravenous drug users, and Haemophilus influenzae is seen among neonates.

An 85-year-old woman comes to the emergency department because of a throbbing right-sided headache for the past 3 hours. She describes exquisite tenderness over her right temple without associated nausea or photophobia. Physical examination shows a pulsating right temporal artery. Which of the following conditions is the patient at an increased risk for developing? A. Brain metastasis B. Hearing loss C. Stroke D. Subarachnoid hemorrhage E. Vision loss

E. Vision loss Giant cell arteritis (GCA) can lead to permanent vision loss. Patients with suspected GCA should be treated empirically with glucocorticoids, even before diagnostic evaluation by temporal artery biopsy is complete.

A 52-year-old man comes to the clinic due to fatigue, joint pain, and recent appearance of skin lesions on his legs. His medical history is noncontributory except for a car accident as a child which required hospitalization and multiple blood transfusions. He has not seen a physician in more than 10 years. The patient does not use tobacco, alcohol or illicit drugs. He is monogamous with one sexual partner. Temperature is 37.0°C (98.6°F), pulse is 75/min, and blood pressure is 130/85 mmHg. Physical exam shows purpuric papules covering the lower extremities that do not blanch with pressure. The rest of the physical examination is unremarkable. Laboratory tests are obtained and shown below: Laboratory value Result Hemoglobin 15 g/dL Leukocyte count 7,000/mm3 Platelets 250,000/mm3 ALT 110 U/L AST 75 U/L Rheumatoid factor Positive Antinuclear antibodies Negative Urinalysis is unremarkable. Serum protein electrophoresis and immunofixation reveals polyclonal IgG and monoclonal IgM bands. Testing for cryoglobulins shows no precipitation after 24 hours. Which of the following tests is most likely to reveal this patient's underlying condition? A. Antibodies to rickettsiae B. Rapid plasma reagin test (RPR) C. Serum anti-neutrophil cytoplasmic antibody (ANCA) D. Bone marrow biopsy E. PCR for N. gonorrhea F. Hepatitis C serology G. Antibodies to citrullinated peptide antigens

F. Hepatitis C serology This patient, who is presenting with purpura, arthralgias, and weakness (Meltzer triad) in association with positive rheumatoid factor test and confirmatory electrophoresis results, has cryoglobulinemia (type II). Mixed cryoglobulinemia syndrome is most often due to chronic hepatitis C infection. The patient's age and history of blood transfusions as a child put him at increased risk for hepatitis C infection. An additional clue is this patient's elevated liver enzymes, suggesting a chronic liver condition. Cryoglobulins are immunoglobulins that precipitate in the cold temperatures and dissolve on rewarming. Cryoglobulins that contain more than one type of immunoglobulin component are called mixed cryoglobulins (e.g., monoclonal/polyclonal IgM, IgG, IgA antibodies, monoclonal κ or λ free light chain portions of these antibodies, and proteins of the blood complement system). The pathogenesis of the condition is poorly understood and assumed to be related to higher production of immunoglobulin, increased precipitation of immune complexes, and insufficient clearance. In type I cryoglobulinemia, precipitates are often seen within 24 hours. However, 3-5 days are usually needed in patients with mixed cryoglobulins, as well as type II cryoglobulins, which requires up to 7 days for precipitation. Clinical manifestations commonly include purpura, arthralgias, weakness and possibly peripheral neuropathy, and renal involvement (e.g. glomerulonephritis).

A previously healthy 36-year-old woman comes to the clinic due to headaches, lightheadedness, diffuse arthralgia, and numbness in the hands for the past week. She also reports fatigue and waking up in the middle of the night "soaked in sweat." She does not smoke cigarettes or use illicit drugs. Temperature is 36.5°C (97.7°F), and blood pressure is 140/95 mmHg in the left arm and 110/65 in the right arm. Physical examination reveals a weak right radial pulse. A systolic bruit is heard over the right subclavian artery. Laboratory results are obtained and shown below: Laboratory value Result Hemoglobin 10.1 g/dL Hematocrit 31% Leukocyte count 12,100 /mm3 Platelet count 550,000/mm3 Erythrocyte sedimentation rate 80 mm/h Serum C-reactive protein 35 mg/dL (N=0.08-3.1) Which of the following is the most likely diagnosis? A. Granulomatosis with polyangiitis B. Polyarteritis nodosa C. Microscopic polyangiitis D. Temporal arteritis E. Thromboangiitis obliterans F. Takayasu arteritis

F. Takayasu arteritis Takayasu arteritis is a large-vessel vasculitis that mainly affects women at ages 10-40 years. One of the classic signs is weak peripheral pulses due to narrowing of the arteries from inflammation, as well as blood pressure discrepancy between affected limbs. Main explanation Takayasu arteritis (TA) is a large-vessel vasculitis that mainly affects the aorta and its branches. It is most often seen in women at ages 10-40 years, especially of Asian descent. Patients initially present with constitutional symptoms (e.g., night sweats, fatigue), followed by symptoms associated with vascular damage. The pathogenesis is poorly understood but appears to occur in a similar pathogenetic mechanism to temporal arteritis. TA is characterized as an inflammatory granulomatous vasculitis of medium- and large-sized arteries, which leads to transmural fibrous thickening of the arterial walls, resulting in multiple vascular obstructions and eventual ischemic changes. Arterial wall inflammation is characterized by a predominance of CD4+ T lymphocytes and macrophages, which undergo granulomatous organization and formation of giant cells. The inflammation induces vascular remodeling that leads to intimal hyperplasia and lumen occlusion. The following features may be seen: Constitutional symptoms: low-grade fever, weight loss, fatigue Cardiovascular: Absent or weak peripheral pulses, limb claudication, blood pressure discrepancy between arms, hypertension, angina, myocardial infarction Musculoskeletal: arthralgias, myalgias Dermatological: skin lesions similar to erythema nodosum or pyoderma gangrenosum Neurologic symptoms, caused by involvement of the carotid and vertebral arteries: lightheadedness, vertigo, syncope, headaches, and strokes Laboratory findings: elevated ESR, CRP, anemia of chronic disease, thrombocytosis and leukocytosis Since TA is primarily an autoimmune condition, initial treatment of symptomatic TA begins with corticosteroids. In more severe conditions, surgical revascularization is a possible therapy as well.

T or F: MTX is safe in pregnancy and in breastfeeding.

FALSE MTX is teratogenic (avoid pregnancy for >3 months for males, >1 olvulatory cycle for females) Breastfeeding: contraindicated

T or F: Xray is always indicated in a patient presenting with low back pain.

FALSE No imaging is necessary unless red flags are present

T of F: It is okay to combine pegloticase with xanthine oxidiase inhibitors such as allopurinol/febuxostat.

FALSE No other XOI should be used while patients are on pegloticase.

Polymyalgia Rheumatica (PMR) (is/is not) ________ associated with specific muscle weakness.

Is not PMR typically is associated with proximal muscle aches and stiffness WITHOUT specific muscle weakness. Seen in those >60 y/o Can be associated with GCA 2-4 weeks duration of sxs ESR >40, anemia, and normal CPK Typically has a rapid response to low dose prednisone

____________ ___________ are the most common benign tumors of the wrist. They are caused by repetitive microtrauma which leads to mucinous degeneration of connective tissue.

Ganglion cysts Tx: Wrist splinting; aspiration w/ steroid injection •Surgical excision for cosmetic reasons

A fracture of the C2 pedicle is called a ____________ fracture.

Hangman's

Xanthine oxidase inhibitors (i.e allopurinol/febuxostat) may (increase/decrease) ___________ concentrations of azathioprine, theophylline, and 6-mercaptopurine.

Increase

Ankylosing spondylitis is a spondyloarthropathy commonly associated with (GI disorder) _____________.

Inflammatory bowel disease

What is the method of action of probenecid?

Inhibition of the reabsorption of urate at the proximal convoluted tubule

_____________ works by inhibiting pyrimidine synthesis via inhibition of dihydroorotate dehydrogenase, resulting in antiproliferative and anti-inflammatory effects.

Leflunomide Prodrug - must be broken down into its active metabolite via liver Undergoes enterohepatic recirculation: -half life 14 to 15 days Requires a loading dose Time to benefit: 4 to 12 weeks (much quicker) ADR's: Common: nausea, diarrhea, rash, alopecia Serious: hepatotoxicity Monitoring: -Screening for pregnancy and TB before initiation of therapy -CBC, serum phosphate, serum transaminases

Anti-centromere Ab's are seen in 70-80% of (diffuse/limited) _____________ scleroderma patients.

Limited

(Diffuse/Limited) __________ scleroderma is associated with CREST.

Limited CREST: Calcinosis Raynauds Esophageal motility Sclerodactyly Telangiectasia

If you think your patient potentially has a tumor, infection, compression from a tumor, or has a hx of prior lumbar surgery, what type of MRI should you order?

MRI w/ and w/o contrast

Leuvocorin is a treatment for inadvertent ___________ overdose.

MTX MOA: supplies cofactor blocked by MTX

Inclusion body myositis most commonly affects (women/men) _____.

Men

Leucovorin is a reversal agent for _____________.

Methotrexate Leucovorin supplies the cofactor blocked by MTX

________ type endplate changes are characterized as bone marrow lesions seen within a vertebral body on magnetic resonance imaging (MRI), suggestive of being associated with low back pain

Modic Modic changes are typically caused by micromotion, bone is becoming inflamed, edema forming.

In what malignant bone tumor do you typically see "punched out" lytic lesions? Patients also present with hypercalcemia due to breakdown of bone.

Multiple Myeloma XR: punched out lytic lesions -bone scans not helpful (b/c lesions are osteoclastic) -labs: anemia, hypercalcemia (break down of bone), renal insufficiency/failure -Bence Jones Protein

In (rheumatoid arthritis/osteoarthritis) ______________ there is DIP involvement (Heberden's nodes) and PIP involvement (Bouchard's nodes).

Osteoarthritis

_______________ is the most common benign bone tumor.

Osteochondroma Seen in 10 to 20 y/o M > F Path: Cartilage-forming tumors -osteocartilaginous exostosis is a bony spur arising on external surface of a bone. HPI: +/- pain, functional problems, deformity, pathologic fx, palpable near ends of long bones Risk of malignant transformation to chondrosarcoma in adults PE: mass, pain, nerve entrapment, myositis Tx: no tx typically, surgical resection possible Osteochondroma mostly affects long bones, pelvis, or shoulder blade.

This type of malignant bone cancer is often characterized by a "sunburst" pattern on Xray. What type of bone cancer is this and what is a common risk factor for this malignancy?

Osteosarcoma Risk: retinoblastoma -> increases risk (mutation in tumor suppressor genes at the Rb oncogene) 90% of mets travel to the lungs MRI is gold standard, along with biopsy to confirm 60% of these are found around the knee Has a bimodal age distribution (M>F)

(Probenecid/Pegloticase) _________ is contraindicated in G6PD deficiency.

Pegloticase

Antihistamines and corticosteroids are required to prevent a hypersensitivity/anaphylactic reaction cause by infusion of (probenacid/pegloticase) ________________.

Pegloticase

______________ converts uric acid to allantoin which is a water soluble metabolite of uric acid. HINT: IV only.

Pegloticase LAST LINE drug for severe gout disease burden + tophi (refractory to ULT) MUST check serum urate level prior to each infusion (common to lose efficacy and develop Ab's) -> gout prophylactic therapy required at least 1 wk prior to infusion and for at least 6 mo after.

What may help you differentiate Takayasu's arteritis from Giant Cell arteritis?

Person's age Takayasu's arteritis is typically seen in women (asian) <40 y/o. GCA is seen more commonly in the elderly

____________ is a monoclonal antibody against CD20 on B-cells which results in B cell depletion.

Rituximab Last line medication, MUST be given with MTX Infusion only -> requires pt to premedicate with methylprednisolone, acetaminophen, and aintihsitamine 30 min prior to infusion Monitor: CBC, hepatitis, renal function, fluid balance Contraception counseling important -> cannot become pregnant for 12 mos following discontinuation

Antinuclear antibody ______ is indicative of diffuse systemic scleroderma.

Scl-70 - Anti-topoisomerase I (scl-70)**Very specific antibodies in 30-40% - RNA polymerase III antibodies 20% - Absence of anti-centromere Ab's

On an oblique posterior-anterior X-ray, vertebral lesions known as spondylolysis are indicated by the ___________ sign with the fracture representing a collar.

Scotty-dog

Inclusion body myositis is characterized by (slow/quick) _______ progressive deterioration of both distal and proximal muscles of the upper and lower extremities.

Slow

The (knee/spine) __________ is the most common site for bone metastases.

Spine Other common sites: hip/pelvis, femur, humerus, ribs, and skull.

Slippage of the vertebral body is defined as _____________.

Spondylolisthesis

_________________ is defined as a defect through the pars interarticularis. What Xray view is best for seeing this condition?

Spondylolysis Oblique view, AKA "Scotty Dog" view.

_______________ is a defect in the pars interarticularis, while _____________ is forward translation of one vertebral body on another.

Spondylolysis......... Spondylolisthesis

___________________ is a cutaneous complication associated with Allopurinol.

Stevens-Johnson Syndrome

While the exact MOA of ___________ is unknown, it is believed that it modulates local chemical mediators of the inflammatory response (esp. leukotrienes).

Sulfasalazine Safe for use in pregnancy (cat B) -caution in breastfeeding Enteric-coated tab formulation preferred (less ADR's) Time to benefit: 1-3 mo ADR's: Common: nausea, dyspepsia, rash, headache, reversible oligospermia (take with meals and break up to meal times) Serious: leukopenia, hemolytic and megaloblastic anemia, hepatotoxicity Monitoring: -CBC w/ diff, LFTs, renal function tests, urinalysis Drug interactions: -many interfere w/ other renally excreted meds

_____________ is known to cause oligospermia, but is safe in pregnancy.

Sulfasalazine Unknown MOA ADR's: -nausea -dyspepsia -rash -headache -reversible oligospermia SERIOUS ADR's: -leukopenia -hemolytic and megaloblastic anemia -hepatotoxicity Safe for pregnancy, but caution in breastfeeding Monitoring: -CBC w/ diff -LFTs -Renal function -Urinalysis

Patients with polymyositis experience proximal (asymmetric/symmetric) muscle weakness.

Symmetric

T or F: Microscopic polyangiitis is noted for it's absence of granuloma formation in the kidney's, lungs, and skin.

TRUE MPA does NOT form granulomas GPA is notable for its granulomatous pulmonary nodules that can cavitate and cause hemorrahge. Remember: MPA is p-ANCA and MPO positive GPA is c-ANCA and PR-3 positive Side note: Eosiniphilic granulomatosis with polyangitis will be p-ANCA and MPO positive OR ANCA may be negative. EGPA can cuase necrotizing granulomas.

Which of the following lab orders is not indicated for a patient when suspecting rheumatoid arthritis? a. LFTs b. CMP, CBC c. ESR/CRP d. RF e. CCP/ACPA

a. LFT's Diagnosis not made on lab values alone, it is based on a combination of hx, PE, and labs.

Which of the following drugs are NOT safe in pregnancy? (multiple answers may apply) a. MTX b. leflunomide c. hydroxychloroquine d. sulfasalazine e. glucocorticoids f. TNF-alpha inhibitors

a. MTX b. leflunomide TNF-alpha inhibitors ARE safe in pregnancy but need to be discontinued prior to 30 weeks

Which of the following drugs may cause drug interactions with MTX? a. omeprazole b. tramadol c. PCN d. cyclobenzaprine

a. omeprazole Others include: -meds that decrease blood flow to the kidney or are renally excreted may inc. MTX concentrations -Ciprofloxacin -NSAIDS (can still use, but low dose for short period of time) -cyclosporine -PCN -PPI (specifically omeprazole) -Loop diuretics

The most common cause of death in scleroderma is sclerosis of pulmonary tissue causing pulmonary __________________.

arterial hypertension

Your patient has been diagnosed with bone metastases but their primary cancer is unknown. What diagnostic imagine test should you use to identify their primary? a. MRI b. CT c. Xray d. Ultrasound

b. CT

Which of the following classes of medications are known to cause potential thrombosis? a. IL-1 inhibitors b. JAK inhibitors c. Non-TNF inhibitors d. TNF-alpha inhibitors

b. JAK inhibitors Examples: -tofacitinib (xeljanz) -baricitinib (olumiant) -upadacitinib (rinvoq) Monitor: CBC, lipids, LFTs, skin exams MOA: Inhibits JAK enzymes which are involved in hematopoiesis and immune cell function DDI: -other biologics -immunosuppressants -CYP3A4 inhibitors Requires renal dose in renal and hepatic disease

Granulomatosis with polyangiitis (GPA) is associated with (P/C) ____ ANCA positive and _____

c-ANCA positive and PR-3 Microscopic polyangitis (MPA) is associated with p-ANCA and MPO

Which of the following is NOT an appropriate treatment option for bone cysts? a. Observation b. Aspiration & injection w/ steroids or bone marrow c. Chemotherapy d. Curettage & bone grafting

c. Chemotherapy All other options may be indicated. (d) is typically only used for larger lesions at risk for pathologic fx Bone cysts are described as well marginated, cystic, purely lytic, moderate expansile lesion Require confirmation w/ biopsy

Your patient presents with Scoliosis, and based on Xray you note a Cobb angle of 50 degrees. Which of the following is your next step in treatment? a. Observation b. Physical therapy c. Surgical eval d. Brace for 16-18 hours per day and exercise

c. Surgical eval Based on Cobb angle <20 routine xray, regular exercise and PT 20-45: brace for 16-18 per day and regular exercise >45: surgical eval

Which of the following drugs is a prodrug, meaning it needs to be broken down into active metabolites vita enterohepatic recirculation? a. sulfasalazine b. hydroxychloroquine c. leflunomide d. MTX

c. leflunomide Inhibits pyrimidine synthesis via inhibition of dihydroorotate dehydrogenase, resulting in antiproliferative and antiinflammatory effects 4-12 wks to benefit Half life: 14-15d Loading nose required Monitor: pregnancy, TB, CBC, serum phosphate, serum transaminases Common: nausea, diarrhea, rash, alopecia Serious: hepatotoxicity Teratogenic (must be undetectable in serum concentrations) DDI: -immunosuppressants -hepatotoxic meds (MTX) DO NOT USE in preexisting liver disease or renal impairment or pregnancy or breastfeeding Elimination procedure: Cholestyramine

A 70-year-old man comes to the office because of aching and stiffness, especially first thing in the morning. It began last month. He indicates that the symptoms occur in the distribution shown here: (shoulders/hips) Strength is 5/5 in all limbs. He has hypertension, and his medications are lisinopril and rosuvastatin. He says that aside from occasional headaches, he feels well. His temperature is 37.5°C (99.5°F), pulse 75/minute, respirations 14/minute, and blood pressure is 140/80. The only abnormal laboratory blood and urine findings are an ESR of 100 mm/hr and hemoglobin of 11 g/dL. Which of the following is the most likely diagnosis? a. fibromyalgia b. multiple myeloma c. polymyalgia rheumatica d. rheumatoid arthritis

c. polymyalgia rheumatica Polymyalgia rheumatica (PMR) is an inflammatory condition that occurs almost exclusively in adults. It is the second most common systemic rheumatic disease after rheumatoid arthritis.Patients present with symmetrical morning stiffness and/or pain in the shoulders, hip girdle, neck, and torso. Weakness is subjective, but decreased range of motion is apparent during physical examination. Patients may also experience systemic symptoms such as malaise, weight loss, and fever.There is an important association between PMR and giant cell (temporal) arteritis. About 50% of patients with temporal arteritis have PMR, and up to 30% of patients with PMR experience symptoms of giant cell arteritis. Diagnosis is based on symptoms and increased ESR and CRP. Normocytic anemia is another common but nonspecific finding. Resolution usually occurs within 3 days of starting a low-dose glucocorticoid, and this response often confirms the diagnosis.

Which of the following is typically the best treatment for asymptomatic lipomas? a. Surgical excision b. Radiation c. ED&C d. Observation

d. Observation MRI indicated if >10cm, rapid growth, pain, fixed, location in deep tissues, or in the thigh.

Which of the following med classes is linked to multiple lipoma formation? a. TNF-alpha inhibitors b. Malaria prophylactics c. NSAIDs d. Protease inhibitors

d. Protease inhibitors


Kaugnay na mga set ng pag-aaral

Prejudice vocab, practice, and quiz

View Set

Amino Acids, Peptides, and Protein Structures

View Set

Bases socioculturales de la conducta humana

View Set

Unit 8.11 Money Laundering (Series 65)

View Set